Unit 3 - Nursing Process, Nursing Process Practice Questions, Chapter 2. Critical Thinking & the Nursing Process, Chapter 3. Nursing Process: Assessment, Chapter 5. Nursing Process: Planning Outcomes, Chapter 6. Nursing Process: Planning Intervention...

¡Supera tus tareas y exámenes ahora con Quizwiz!

What do standardized nursing care plans and individualized care plans have in common? They both 1) Reflect critical thinking for a specific patient 2) Are preprinted to apply to needs common to a group of patients 3) Address a patient's individual needs 4) Provide detailed nursing interventions

NS: 4 They both provide detailed nursing interventions, although the individualized care plan is more specific to the patient's needs and reflects critical thinking, whereas standardized plans do not. It is not true of individual nursing care plans that they are preprinted and apply to a group. PTS: 1 DIF: Moderate REF: pp. 87 KEY: Nursing process: Planning | Client need: SECE | Cognitive level: Analysis

Match the assessment technique to the data that should be collected. There may be more than one technique used to collect the data. 1) Auscultation 2) Inspection 3) Palpation 4) Percussion ____ 1. Skin pink, warm, and dry ____ 2. Lung sounds clear ____ 3. Abdomen is tympanic ____ 4. Abdomen soft and nontender

1. ANS: 2 PTS: 1 DIF: Moderate REF: p. 42 KEY: Nursing process: Assessment | Client need: PHSI | Cognitive level: Application 2. ANS: 1 PTS: 1 DIF: Moderate REF: p. 42 KEY: Nursing process: Assessment | Client need: PHSI | Cognitive level: Application 3. ANS: 4 PTS: 1 DIF: Moderate REF: p. 42 KEY: Nursing process: Assessment | Client need: PHSI | Cognitive level: Application 4. ANS: 3 PTS: 1 DIF: Moderate REF: p. 42 KEY: Nursing process: Assessment | Client need: PHSI | Cognitive level: Application

Match the critical thinking attitude on the left with the appropriate example on the right. 1) Reading the instruction manual of a new glucose monitoring machine 2) Asking for help with a procedure because you have not done it before 3) Obtaining the latest research about a new diagnostic procedure even though the articles are difficult to find 4) Questioning the reason for a new staffing policy 5) Realizing your feelings about alternative medicine may interfere with the care you give a patient 6) Asking a patient's feelings about his cancer diagnosis 7) Questioning your feelings when a patient's family requests withholding nutrition for a terminally ill client ____ 1. Independent thinking ____ 2. Intellectual curiosity ____ 3. Intellectual humility ____ 4. Intellectual empathy ____ 5. Intellectual courage ____ 6. Intellectual perseverance

1. ANS: 4 PTS: 1 DIF: Difficult REF: p. 26 KEY: Nursing process: N/A | Client need: SECE | Cognitive level: Application 2. ANS: 1 PTS: 1 DIF: Difficult REF: p. 26 KEY: Nursing process: N/A | Client need: SECE | Cognitive level: Application 3. ANS: 2 PTS: 1 DIF: Difficult REF: p. 26 KEY: Nursing process: N/A | Client need: SECE | Cognitive level: Application 4. ANS: 6 PTS: 1 DIF: Difficult REF: p. 26 KEY: Nursing process: N/A | Client need: SECE | Cognitive level: Application 5. ANS: 7 PTS: 1 DIF: Difficult REF: p. 26 KEY: Nursing process: N/A | Client need: SECE | Cognitive level: Application 6. ANS: 3 PTS: 1 DIF: Difficult REF: p. 26 KEY: Nursing process: N/A | Client need: SECE | Cognitive level: Application

Match the type of special needs assessment with the correct example. 1) Do you perform monthly breast self-exams? 2) Do you live near any industrial manufacturing plants? 3) Who can you talk to when you feel sad? 4) Who do you live with? 5) What is your understanding of your diet? 6) Since you had your stroke, have you had any problems dressing yourself? 7) Do you have a religious preference? ____ 5. Community ____ 6. Family ____ 7. Functional ability ____ 8. Nutrition ____ 9. Psychosocial ____ 10. Wellness ____ 11. Spiritual

5. ANS: 2 PTS: 1 DIF: Easy REF: pp. 43-44 KEY: Nursing process: Assessment | Client need: SECE | Cognitive level: Application 6. ANS: 4 PTS: 1 DIF: Easy REF: pp. 43-44 KEY: Nursing process: Assessment | Client need: SECE | Cognitive level: Application 7. ANS: 6 PTS: 1 DIF: Easy REF: pp. 43-44 KEY: Nursing process: Assessment | Client need: SECE | Cognitive level: Application 8. ANS: 5 PTS: 1 DIF: Easy REF: pp. 43-44 KEY: Nursing process: Assessment | Client need: SECE | Cognitive level: Application 9. ANS: 3 PTS: 1 DIF: Easy REF: pp. 43-44 KEY: Nursing process: Assessment | Client need: SECE | Cognitive level: Application 10. ANS: 1 PTS: 1 DIF: Easy REF: pp. 43-44 KEY: Nursing process: Assessment | Client need: SECE | Cognitive level: Application 11. ANS: 7 PTS: 1 DIF: Moderate REF: pp. 43-44 KEY: Nursing process: Assessment | Client need: SECE | Cognitive level: Recall

Match the terms from the critical thinking model in your text with the correct example. 1) I wonder if my values about quality of life have affected my thinking. 2) What should I have done differently? 3) I need to talk with the client to make sure the family gave me the correct information. 4) I have been through a situation like this before. 5) There are several interventions that would work in this situation. 6) I need to follow the steps in the procedure manual. ____ 7. Contextual awareness ____ 8. Inquiry ____ 9. Considering alternatives ____ 10. Analyzing assumptions ____ 11. Reflecting skeptically

7.ANS:4PTS:1DIFgrinifficult REF: p. 28-29; High-level question, answer not stated verbatim KEY: Nursing process: N/A | Client need: SECE | Cognitive level: Analysis 8.ANS:3PTS:1DIFgrinifficult REF: p. 28-29; High-level question, answer not stated verbatim KEY: Nursing process: N/A | Client need: SECE | Cognitive level: Analysis 9.ANS:5PTS:1DIFgrinifficult REF: p. 28-29; High-level question, answer not stated verbatim KEY: Nursing process: N/A | Client need: SECE | Cognitive level: Analysis 10.ANS:1PTS:1DIFgrinifficult REF: p. 28-29; High-level question, answer not stated verbatim KEY: Nursing process: N/A | Client need: SECE | Cognitive level: Analysis 11.ANS:2PTS:1DIFgrinifficult REF: p. 28-29; High-level question, answer not stated verbatim KEY: Nursing process: N/A | Client need: SECE | Cognitive level: Analysis

A nursing student expresses some confusion about identifying the appropriate nursing diagnosis for a specific client. Which of the following responses by the clinical instructor is most accurate and helpful? ="After defining the client's symptomatology, eliminate those nursing diagnoses that are not supported by the database." ="Assess your client and then select the nursing diagnosis that has the greatest number of observable defining characteristics." ="After assessing the client, compare their symptoms carefully to the defining characteristic of the nursing diagnosis in order to support or eliminate it as applicable." ="With experience you will become skilled at identifying the defining characteristics of a nursing diagnosis in your client. Until that time use a nursing diagnosis book to help in the selection process."

="After assessing the client, compare their symptoms carefully to the defining characteristic of the nursing diagnosis in order to support or eliminate it as applicable." After assessing the client, always examine the defining characteristics in your database carefully to support or eliminate a nursing diagnosis. Although the other options are correct, they do not provide as concise an explanation as "after assessing the client, compare their symptoms carefully to the defining characteristic of the nursing diagnosis in order to support or eliminate it as applicable."

The nurse works with the respiratory therapist to administer a patient's breathing treatments. He reports the patient's breathing status and tolerance of the treatment to the primary care provider. The nurse then discusses with the patient the options for further treatment. This is an example of: =Collaboration =Delegation =Coordination of care =Supervision of care

=Collaboration Collaboration means working with other caregivers to plan, make decisions, and perform interventions. Delegation is the transfer to another person of the authority to perform a task in a selected situation. Coordination of care involves scheduling treatments and activities with other departments, putting together all the patient data to obtain "the big picture." Supervision is the process of directing, guiding, and influencing the outcome of an individual's performance of an activity or task.

A nursing instructor asked his nursing students to discuss their experiences with charting assessment data. Which comment by the student indicates the need for further teaching? (This means, "which answer is wrong?") ="I find it difficult to avoid using phrases like 'the patient tolerated the procedure well.'" ="It's confusing to have to remember which abbreviations this hospital allows." ="I need to work on charting assessments and interventions right after they are done." ="My patient was really quiet and didn't say much, so I charted that he acted depressed."

="My patient was really quiet and didn't say much, so I charted that he acted depressed." When charting data, chart only what was observed, not what it meant. Inferences should not be made about a patient's behavior during data collection ("he acted depressed"), so that response reflects the student's lack of knowledge and need for teaching. (Instead, the student would chart something like "The patient would not make eye contact and remained facing the window. Answered with only I don't know to all questions.) Chart specific data, not vague phrases; the student is acknowledging the importance of this. There are no universally accepted phrases, just agency-approved abbreviations; the student is acknowledging the need to use agency-approved abbreviations. The student is correct that charting should be completed as soon after data collection as possible.

The nurse recognizes that expected client outcomes should be documented according to specific criterion in order that they are clear and easily understood by other members of the health care team. Of the following, the outcome statement that best meets the established criteria is the following: ="Vital signs will return to within normal levels for a middle aged adult." ="Nursing assistant will ambulate the client in the hallway 3 times each day." ="Lungs will be clear to auscultation and respiratory rate will be 20/minute." ="Output will be at least 100 mL/hour of clear yellow urine within 24 hours."

="Output will be at least 100 mL/hour of clear yellow urine within 24 hours." "Output will be at least 100 mL/hour of clear yellow urine within 24 hours." is client-centered, singular, observable, measurable, time-limited, and realistic. "Vital signs will return to within normal levels for a middle aged adult." is not measurable (i.e., guidelines for normal are not stated), and it is not time-limited (e.g., by when?). "Nursing assistant will ambulate the client in the hallway 3 times each day." is not client-centered. "Lungs will be clear to auscultation and respiratory rate will be 20/minute." is not singular and it is not time-limited.

The nurse decides to interview the client using the open-ended question technique. Which of the following statements reflects this type of questioning? ="Is your pain worse or better than it was an hour ago?" ="Do you believe that your nausea is from the new antibiotic?" ="What do you think has been causing your current depression?" ="What have you done to alleviate the side effects from your medications?"

="What do you think has been causing your current depression?" An open-ended question prompts the client to describe a situation in more than one or two words. This option demonstrates the open-ended question technique. This question limits the client's answers to one or two words. It is an example of a closed-ended question. The question in this option limits the client's answer to one or two words such as "yes" or "no." It is an example of a closed-ended question. This option only requires a few words to form an answer. It does not use the open-ended question technique.

A nursing instructor is guiding nursing students on best practices for interviewing patients. Which of the following comments by a student would indicate the need for further instruction? ="My patient is a young adult, so I plan to talk to her without her parents in the room." ="Because my patient is old enough to be my grandfather, I will call him Mr." ="When reading my patient's health record, I thought of a few questions to ask." ="When I give my patient his pain medication, I will have time to ask questions."

="When I give my patient his pain medication, I will have time to ask questions." A patient should be comfortable when interviewing. The pain medication should have time to work before the nurse would consider interviewing the patient, so asking questions when giving the medication is not a good idea. It is appropriate to interview patients without family/friends around. In nearly every culture, calling a patient Mr. or Mrs. shows respect and is, therefore, correct. Reading the patient's health record is appropriate preparation for an interview.

Nursing interventions may be categorized based upon the degree of nursing autonomy. Which of the following nursing interventions is considered as physician- or prescriber-initiated? =Teaching a client to administer his or her insulin injection =Assisting a new mother with learning the art of breast-feeding =Notifying the nutritionist of a client's specific dietary preferences =Administering a cleansing enema in preparation for radiological testing

=Administering a cleansing enema in preparation for radiological testing Preparing a client for a diagnostic test is an example of a physician-initiated intervention. Teaching a client to administer his or her insulin injection is an example of a nurse-initiated intervention. Assisting a new mother with breast-feeding is an example of a nurse-initiated intervention. Notifying a nutritionist of a client's dietary preferences is a collaborative intervention.

The certified nursing assistant (CNA) is feeding a patient and notes that the patient is having difficulty swallowing. She reports this to the primary registered nurse. What should the nurse do first? =Assign the task to a more experienced CNA =Feed the patient herself =Assess the patient and place on NPO status =Call the primary care provider

=Assess the patient and place on NPO status Feeding a patient is a delegatable task that a CNA can perform. However, once it is reported to the registered nurse that the patient is having difficulty swallowing, this becomes a safety issue that the registered nurse must address. This circumstance is then no longer delegatable for any CNA regardless of experience. The first action by the nurse is to assess the patient and place the patient on NPO status until a primary provider is notified for further orders.

A nurse seeks to organize the data obtained from the client in a logical manner. The organizational method that identifies relationships between factors and symptoms in the database is known as: =Clustering data =Validating data =Peer reviewing =Problem statement

=Clustering data Clustering data means the nurse organizes the information obtained into meaningful clusters. A cluster is a set of signs or symptoms grouped together in a logical order. When clustering data, the nurse identifies relationships between factors and symptoms. Validating data means to compare the data obtained with another source to ensure its accuracy. Peer review is the evaluation of the quality of the work effort of an individual by his or her peers. After validating data and clustering data, the nurse may formulate a problem statement, usually in the form of a nursing diagnosis.

Of the following statements, which one is an example of an appropriately written nursing diagnosis? =Acute pain related to left mastectomy =Impaired gas exchange related to altered blood gases =Deficient knowledge related to understanding of need for cardiac catheterization =Need for high protein diet related to alteration in client nutrition

=Deficient knowledge related to understanding of need for cardiac catheterization The third example is a nursing diagnosis that is written correctly. It defines a problem and its etiology. In this case the problem is the client's response to a diagnostic test. A medical diagnosis should not be recorded as the etiology because nursing interventions cannot change the medical diagnosis. It would be appropriate to state acute pain related to impaired skin integrity secondary to mastectomy incision. This nursing diagnosis is written incorrectly because it uses supportive data of the problem as the etiology. This nursing diagnosis does not identify the problem and etiology. It identifies the client's goal rather than the problem. It could be reworded as imbalanced nutrition: less than body requirements related to inadequate protein intake.

Which of the following describes the most important use of making a nursing diagnosis? Assume all are true. =Differentiates the nurse's role from that of the physician =Identifies a body of knowledge unique to nursing =Helps nursing develop a more professional image =Describes the client's problem and needs for nursing care

=Describes the client's problem and needs for nursing care The benefits to nurses and nursing are that nursing diagnoses differentiate the nurse's role, they identify a unique body of nursing knowledge, and some think they help nursing to develop a more professional image. However, the primary goal of nursing is to serve the good of the patient. Therefore, the most important use of a diagnosis is to specifically identify the client's needs for quality nursing care.

The nurse is individualizing Mr. Wu's plan of care by writing a plan for his nursing diagnosis of Anxiety. The nurse needs to write outcomes on the plan of care because outcomes describe =Desirable changes in the patient's health status =Specific patient responses to medical interventions =Specific nursing behaviors to improve a patient's health =Criteria to evaluate the appropriateness of a nursing diagnosis

=Desirable changes in the patient's health status Outcomes describe changes in the patient's health status in response to nursing rather than medical interventions. Outcomes relate to patient behavior, not nursing behaviors. Outcomes are a measure of the effectiveness of nursing care for a specific nursing diagnosis, not whether the nursing diagnosis is appropriate.

The primary purpose of the nursing evaluation process is to: =Determine the effectiveness of the nursing care provided =Identify interventions that are ineffective in achieving client goals =Establish the progress the client is making towards health and wellness =Critique the nurse's ability to implement appropriate nursing interventions

=Determine the effectiveness of the nursing care provided The evaluation process determines the effectiveness of nursing care. The remaining options are all examples of evaluation but do not reflect the primary purpose of nursing evaluation.

The nurse is caring for a client who was newly diagnosed with type 2 diabetes mellitus. Which intervention by the nurse best promotes client cooperation with the treatment plan? =Teaching the client that he must lose weight to control his blood sugar =Informing the client that he must exercise at least three times per week =Explaining to the client that he must come to the diabetic clinic weekly =Determining the client's main concerns about his diabetes

=Determining the client's main concerns about his diabetes Determining the client's main concerns promotes cooperation with the treatment regimen. For example, if the client is concerned about paying for diabetic monitoring equipment, he may disregard any teaching about the procedure. Although it is often important for a diabetic client to exercise and lose weight to control blood sugar levels, the client must want to do both. He will not exercise or lose weight simply because he is told to do so. The nurse must assess the client's support systems and resources, not merely tell him he must come to the diabetic clinic weekly. Some clients do not have access to transportation and, therefore, could not come to the clinic without social service intervention. Remember that knowledge does not necessarily change behavior.

Of the following statements, which one is an example of an appropriately written nursing diagnosis? =Diarrhea related to food intolerance =Alteration in comfort related to pain =Risk for impaired skin integrity related to poor hygiene habits =Potential complications related to insufficient vascular access

=Diarrhea related to food intolerance Diarrhea related to food intolerance is a correctly written nursing diagnosis. It consists of a problem related to an etiology, and it is a condition that nursing interventions can treat or manage. Alteration in comfort related to pain is not written correctly because it is a circular statement. It would be appropriate to state ineffective breathing pattern related to incisional pain. Risk for impaired skin integrity related to poor hygiene habits is not written correctly because it uses a nurse's prejudicial judgment. It would be more appropriate and professional to state risk for impaired skin integrity related to knowledge about perineal care. Potential complications related to insufficient vascular access is not written appropriately because it identifies a nursing problem, not a client's problem. It would be appropriate to state risk for infection related to presence of invasive lines.

When should the nurse make observations about a patient? =When the patient has specific complaints =With the first assessment of the shift =Each time the nurse gives medications to the patient =Each time the nurse interacts with the patient

=Each time the nurse interacts with the patient The nurse should make observations about the patient each time she enters the room or interacts with the patient to gain ongoing data about the patient.

Nursing interventions should be documented according to specific criteria in order that they may be clearly understood by other members of the nursing team. The intervention statement "Nurse will apply warm, wet soaks to the patient's leg while awake" lacks which of the following components? =Method =Quantity =Frequency =Performing staff

=Frequency The intervention statement does not include how frequently the warm soaks should be applied. The method is applying warm, wet soaks to the patient's leg while awake. The quantity is warm, wet soaks. The qualification of the person who will perform the action is the designation of "the nurse."

A nurse is observed conducting an assessment interview for a newly admitted client. Which of the following would require immediate follow-up by the nurse's mentor? =Conducting the interview with the client's boyfriend present =Stopping the interview to answer a page from the nursing station =Frequently checking the time while waiting for the client to answer =Heard asking the client, "Am I correct; you've rated your pain a 9 out of 10?"

=Frequently checking the time while waiting for the client to answer Clients are less likely to fully reveal the nature of their health care problems when nurses show little interest, appear rushed, or are easily distracted by activities around them. As long as the nurse had the client's permission, this would not require follow-up. While interrupting an assessment is not recommended, a page is an example of an acceptable exception and so this would not require follow-up. If the nurse were confirming the information, it would not require follow-up. If the mentor felt the nurse was questioning the validity of client's pain rating, a follow-up would be appropriate because a client's pain rating should not be questioned.

The nurse is concerned that atelectasis may develop as a postoperative complication. Which of the following is an appropriate diagnostic label for this problem, should it occur? (FYI: Atelectasis is the collapse of alveoli.) =Impaired gas exchange =Decreased cardiac output =Ineffective airway clearance =Impaired spontaneous ventilation

=Impaired gas exchange A potential etiology for impaired gas exchange may be atelectasis. Atelectasis would not support the diagnostic label for decreased cardiac output. Atelectasis would not be an etiology for ineffective airway clearance. Increased tenacious sputum production would be a possible etiology for ineffective airway clearance. Impaired spontaneous ventilation would not be an appropriate diagnostic label for atelectasis. Previous Next

Which one of the following is an appropriate etiology (related to) for a nursing diagnosis? =Myocardial infarction =Cardiac catheterization =Abnormal blood gas levels =Increased airway secretions

=Increased airway secretions Increased airway secretions is a condition that responds to nursing interventions and therefore would be an appropriate etiology for a nursing diagnosis. Myocardial infarction would not be an appropriate etiology for a nursing diagnosis because it is a medical diagnosis. Nursing interventions will not alter the medical diagnosis of myocardial infarction. Cardiac catheterization is a diagnostic procedure and would not be an appropriate etiology for a nursing diagnosis. Rather, the client's response to the procedure would be the area of nursing concern. Abnormal blood gas levels would not be an appropriate etiology for a nursing diagnosis because it is not a causative factor, but rather it is a defining characteristic of a problem.

Which of the following is an example of a problem that nurses can treat independently? =Hemorrhage =Nausea =Fracture =Infection

=Nausea A nursing diagnosis (or nursing problem) is a human response to a disease, injury, or other stressor that nurses can identify, prevent, or treat independently. Nausea is the only problem that meets that criterion; all others are medical or collaborative problems.

Which of the following is the most important reason for nurses to be critical thinkers? =Nurses need to follow policies and procedures. =Nurses work with other healthcare team members. =Nurses care for clients who have multiple health problems. =Nurses have to be flexible and work variable schedules.

=Nurses care for clients who have multiple health problems. Critical thinking is essential for client care, particularly when the care is complex, involving numerous health issues. Following policies and procedures does not necessarily require critical thinking; working with others or being flexible and working different schedules do not necessarily require critical thinking

The Joint Commission requires which type of assessment to be performed on all patients? =Functional ability =Pain =Cultural =Wellness

=Pain The Joint Commission requires that pain and nutrition assessment be performed on all patients. Other special needs assessments should be performed when cues indicate there are risk factors.

The nurse is using electronic care planning. He enters the patient's nursing diagnosis into the computer and chooses desired outcomes. He has validated his data, diagnosis, and goals. When he considers the list of interventions that the program generates, he sees that none of them fits this patient's individual needs. What should the nurse do? =Reject them all and type in appropriate interventions. =Select the interventions from the program that are most suitable. =Ask another nurse to assess the patient and give her recommendation =Restart the computer; it is probably a program malfunction.

=Reject them all and type in appropriate interventions. The nurse can reject all the suggested interventions if they do not address patient needs. Nearly all computer programs have a screen that allows you to type in interventions and nursing orders. It is the nurse's responsibility to choose interventions: He cannot abdicate this responsibility and let the computer "choose." As a professional, this nurse has already validated the data, nursing diagnosis, and goals, so he can feel reasonably certain that there is nothing wrong with the plan to that point. Although consultation with other nurses may be a wise and prudent step to take at times, the nurse caring for the patient would likely have the most familiarity with the healthcare needs and is in a better position to make sound judgments than another nurse who does not know the patient. Therefore, it might not be productive or efficient to consult another nurse or restart the computer.

A client is admitted to the hospital with an acute episode of chronic obstructive pulmonary (lung) disease. The nurse makes a diagnosis of Ineffective Breathing Pattern related to inability to maintain adequate rate and depth of respirations and has recorded the diagnosis and appropriate expected outcomes on the care plan. When selecting nursing interventions, what should the nurse do first? =Identify several interventions likely to achieve the desired outcomes. =Review the problem and etiology of the nursing diagnosis. =Choose the best interventions for the patient. =Review the outcomes she has written.

=Review the problem and etiology of the nursing diagnosis. The process of choosing interventions is first to review the nursing diagnosis and etiology; then review the desired outcomes; identify several interventions or actions; choose the best interventions for the patient; and then individualize standardized interventions to meet the patient's unique needs.

Which one of the following is a NANDA International nursing diagnosis label? =Frequent urination =Coughing and dyspnea =Risk for impaired parenting =Abnormal hygienic care practices

=Risk for impaired parenting "Frequent urination" is a symptom, not a NANDA International nursing diagnosis label. "Coughing and dyspnea" are symptoms, not a NANDA International nursing diagnosis label. "Risk for impaired parenting" is a NANDA International nursing diagnosis label. "Abnormal hygienic care practices" is not a NANDA International nursing diagnosis label. It incorrectly implies a nurse's prejudicial judgment.

Care plans created by nursing students usually differ from those that are completed by nurses working on client units. An aspect of the plan that is usually included in the student's care plan but not in the client's record is: =Client outcomes =Nursing diagnoses =Scientific rationales =Nursing interventions

=Scientific rationales An aspect of a nursing care plan that is usually included in the student's care plan, but not in the client's record, is scientific rationales. Client outcomes are included in both student care plans and the client's record. Nursing diagnoses are included in both student care plans and the client's record. Nursing interventions are a component of both student care plans and a nursing care plan in the client's record.

Which of the following characteristics are considered guidelines for the writing of appropriate outcomes? (Select all that apply.) =Singular =Realistic =Practical =Observable =Measurable =Meaningful

=Singular =Realistic =Observable =Measurable

What do critical thinking and the Nursing Process have in common? =They are both linear processes used to guide one's thinking. =They are both thinking methods used to solve a problem. =They both use specific steps to solve a problem. =They both use similar steps to solve a problem.

=They are both thinking methods used to solve a problem. Critical thinking and the Nursing Process are ways of thinking that can be used in problem-solving (although critical thinking can be used for other than problem-solving applications). Neither method of thinking is linear. The Nursing Process has specific steps; critical thinking does not.

The nurse notes that a narcotic is to be administered "per epidural cath." The nurse; however, does not know how to perform this procedure. Which aspect of the implementation process should be followed? =seek assistance =reassess the client =use interpersonal skills =critical decision making

=seek assistance If a nurse does not know how to perform a procedure, he or she should seek assistance. Information about the procedure is obtained from the literature and the agency's procedure book. All equipment necessary for the procedure is collected. Finally, another nurse who has completed the procedure correctly and safely provides assistance and guidance. Reassessing the client is a partial assessment that may focus on one dimension of the client or on one system. Interpersonal skills are used to develop a trusting relationship, express a level of caring, and communicate clearly with the client, family, and health care team. Critical decision making is used when the nurse implements the care plan using the knowledge bases necessary for care planning and then completing the planned interventions most effectively.

The nurse is most likely to collect timely, specific information by asking which of the following questions? A. "Would you describe what you are feeling?" B. "How are you today?" C. "What would you like to talk about?" D. "Where does it hurt?"

A. "Would you describe what you are feeling?" Rationale: This is an open-ended question that will elicit subjective data. The data collected will reflect the client's current health status and human response(s) and should generate specific information that can be used to identify actual and/or potential health problems. Options 2 and 3 are more likely to elicit general, nonspecific information. Option 4 may result in a brief, one-word response or nonverbal gesture indicating the site of the client's pain. A better approach to collect specific information might be, "Describe any pain you are having."

Which of the following behaviors by the nurse demonstrates that the nurse is participating in critical thinking? Select all that apply. A. Admitting not knowing how to do a procedure and requesting help B. Using clever and persuasive remarks to support an opinion or position C. Accepting without question the values acquired in nursing school D. Finding a quick and logical answer, even to complex questions E. Gathering three assistants to transfer the client to a stretcher after noting the client weighs 300 lbs.

A. Admitting not knowing how to do a procedure and requesting help E. Gathering three assistants to transfer the client to a stretcher after noting the client weighs 300 lbs. Rationale: Critical thinking in nursing is self-directed, supporting what nurses know and making clear what they do not know. It is important for nurses to recognize when they lack the knowledge they need to provide safe care for a client (option 1). Nurses must also utilize their resources to acquire the support they need to care for a client safely (option 5). Options 2, 3, and 4 do not demonstrate critical thinking.

The nurse assigned to care for a postoperative client has asked an unlicensed assistive person (UAP) to help the client ambulate in the hall. Before delegating this task, the nurse must do which of the following? A. Assess the client to be sure ambulation with assistance is an appropriate care measure B. Ask the client if he or she is ready to ambulate C. Ask whether the UAP has time to assist the client D. Ask the charge nurse whether UAPs have ambulated the client during this shift

A. Assess the client to be sure ambulation with assistance is an appropriate care measure Rationale: Prior to delegating any client care responsibilities, the nurse must assess the client to assure that the delegation is appropriate to his or her care. Options 2, 3, and 4 would not constitute an assessment of the client's current status.

A client comes to the walk-in clinic with reports of abdominal pain and diarrhea. While taking the client's vital signs, the nurse is implementing which phase of the nursing process? A. Assessment B. Diagnosis C. Planning D. Implementation

A. Assessment Rationale: The first step in the nursing process is assessment, the process of collecting data. All subsequent phases of the nursing process (options 2, 3, and 4) rely on accurate and complete data.

The nurse decides it would be beneficial to the client to allow the client's infant granddaughter to visit before the client's scheduled heart transplant. Before implementing this intervention the nurse should collaborate with which of the following? Select all that apply. A. Client and Family B. Other nursing staff on the unit C. Security department D. Hospital administration E. This is not a collaborative intervention so no collaboration will be needed prior to implementation

A. Client and Family B. Other nursing staff on the unit Rationale: Collaboration with the client and family will encourage a sense of autonomy and active involvement in the healthcare process for the client. In this case collaboration with other nursing staff will ensure the successful implementation of the planned intervention. There is no real need for collaboration with hospital administration or the security department in this situation although the nurse should be aware of her responsibility to collaborate at those levels when the situation demands it.

The nurse would make which of the following inferences after performing the appropriate client assessment? A. Client is hypotensive B. Respiratory rate of 20 breaths per minute C. Oxygen saturation of 95% D. Client relays anxiety about blood work

A. Client is hypotensive Rationale: An inference is the nurse's judgment or interpretation of cues such as judging a blood pressure to be lower than normal. A cue is any piece of data information that influences a decision. Options 2, 3, and 4 are cues that could lead to inferences.

When evaluating an elderly client's blood pressure (BP) of 146/78 mmHg, the nurse does which of the following before determining whether the BP is normal or represents hypertension? A. Compare this reading against defined standards B. Compare the reading with one taken in the opposite arm C. Determine gaps in the vital signs in the client record D. Compare the current measurement with previous ones

A. Compare this reading against defined Rationale: Analysis of the client's BP requires knowledge of the normal BP range for an older adult. The nurse compares the client's data against identified standards to determine whether this reading is normal or abnormal. Measuring the BP in the other arm (option 2) and comparing the reading to previous ones (option 4) will give additional client data, but the comparison alone will not determine whether the BP is normal. Gaps in the record (option 3) will not aid in interpreting the current measurement.

The nurse notes that the client often sighs and says in a monotone voice, "I'm never going to get over this." When encouraged to participate in care, the client says, "I don't have the energy." The nurse believes these cues are suggestive of which nursing diagnoses? Select all that apply. A. Hopelessness B. Powerlessness C. Interrupted sleep pattern D. Disturbed self esteem E. Self care deficit

A. Hopelessness B. Powerlessness Rationale: Rationale: A nursing diagnosis is a clinical judgment about a response to an actual or potential health problem. This client is manifesting symptoms of both hopelessness and powerlessness. Although the client does report symptoms compatible with fatigue, there is no direct data is given that indicates the client has interrupted sleep patterns (option 3), disturbed self esteem (option 4), or self care deficit (option 5).

The nurse suspects that a client is withholding health-related information out of fear of discovery and possible legal problems. The nurse formulates nursing diagnoses for the client carefully, being concerned about a diagnostic error resulting from which of the following? A. Incomplete data B. Generalize from experience C. Identifying with the client D. Lack of clinical experience

A. Incomplete data Rationale: To collect data accurately, the client must actively participate. Incomplete data can lead to inappropriate nursing diagnosis and planning. The other options are not relevant to the question as presented.

During which part of the client interview would it be best for the nurse to ask, "What's the weather forecast for today?" A. Introduction B. Body C. Closing D. Orientation

A. Introduction Rationale: Asking about the weather initiates the social or introductory phase of the interview and allows the nurse to begin an assessment of the client's mental status. The goal is to develop rapport with the client at the beginning of the interview. In the body the client responds to the nurse's questions. During the closing the nurse or the client terminates the interview.

After instructing the client on crutch walking technique, the nurse should evaluate the client's understanding by using which of the following methods? A. Return demonstration B. Explanation C. Achievement of 90 on written test D. Have client explain produce to the family

A. Return demonstration Rationale: Interpersonal skills are the sum of the activities the nurse uses when communicating with others. Technical/psychomotor skills are "hands-on" skills, which are often procedures and are evaluated by return demonstration. Cognitive skills are the intellectual skills of analysis and problem-solving and are evaluated by tests.

The client states, "My chest hurts and my left arm feels numb." The nurse interprets that this data is of which type and source? A. Subjective data from a primary source B. Subjective data from a secondary source C. Objective data from a primary source D. Objective data from a secondary source

A. Subjective data from a primary source Rationale: The client states, "My chest hurts and my left arm feels numb." The nurse interprets that this data is of which type and source?

Which activity would be appropriate for the nurse to delegate to an unlicensed assistive person (UAP)? A. Taking vital signs of clients on the nursing unit B. Assisting the physician with an invasive procedure C. Adjusting the rate on an infusion pump D. Evaluating achievement of client outcome goals

A. Taking vital signs of clients on the nursing unit Rationale: Part of the professional nurse's role is to delegate responsibility for activities while maintaining accountability. The nurse must match the needs of the client with the skills and knowledge of UAPs. Certain skills and activities, such as those in options 2, 3, and 4, are not within the legal scope of practice for a UAP.

Which of these is a correctly stated outcome goal written by the nurse? A. The client will walk 2 miles daily by March 19 B. The client will understand how to give insulin by discharge C. The client will regain their former state of health by April 1 D. The client achieve desired mobility by May 7

A. The client will walk 2 miles daily by March 19 Rationale: Outcome goals should be SMART, i.e., Specific, Measurable, Appropriate, Realistic, and Timely. Option 1 is the only outcome that has a specific behavior (walks daily), with measurable performance criteria (2 miles), and a time estimate for goal attainment (by March 19).

What is wrong with the following diagnostic statement? "Impaired Physical Mobility related to laziness and not having appropriate shoes." The statement is 1) Judgmental 2) Too complex 3) Legally questionable 4) Without supportive data

ANS: 1 "Lazy" implies criticism of the client and therefore is judgmental. There need to be several (certainly more than two) etiological factors for the statement to be complex. There is no blame implied or harm resulting, so the statement is not legally questionable. There is no minimum "amount" of supportive data for a diagnosis and the stated etiology related to the nursing diagnosis. No supportive data are given in the stem of the question, so you could not choose "lack of data" as the best answer because all the options lack data as far as you can tell from the information given in the question. In addition, it is not necessary to include supportive data in the diagnostic statement (although some do prefer to use "A.M.B." and include defining characteristics). PTS:1DIF:ModerateREF:p. 74 KEY: Nursing process: Diagnosis | Client need: SECE | Cognitive level: Analysis

Which of the following is an example of a cluster of related cues? 1) Complains of nausea and stomach pain after eating 2) Has a productive cough and states stools are loose 3) Has a daily bowel movement and eats a high-fiber diet 4) Respiratory rate 20 breaths/min, heart rate 85 beats/min, blood pressure 136/84

ANS: 1 A cue is an unhealthy response; a cluster of cues consists of cues related to each other. Productive cough and loose stools are abnormal findings but are not obviously or usually related to each other. Daily bowel movement and high-fiber diet are related but normal responses. The vital signs are also within normal limits. PTS:1DIFgrinifficultREF: pp. 62 KEY: Nursing process: Diagnosis | Client need: PHSI | Cognitive level: Analysis

A patient underwent surgery 3 days ago for colorectal cancer. The patient's critical pathway states that he should participate in a teaching session with the wound ostomy nurse to learn colostomy self-care. The patient appears depressed and refuses to look at the colostomy or even make eye contact. How should the nurse proceed? 1) Postpone the teaching session until the patient is more receptive. 2) Follow the critical pathway for patient teaching about ostomy care. 3) Administer a prescribed antidepressant and notify the physician. 4) Explain to the patient the importance of skin care around the ostomy site.

ANS: 1 A depressed affect and poor eye contact likely indicate that the client is having difficulty coping with the new colostomy. At this time, the client would not be physically and psychologically ready to obtain the most benefit from teaching pertaining to ostomy care. Therefore, the nurse should postpone the teaching session for this client until the client is receptive to receiving the information. The nurse should not perform the teaching session simply because the critical pathway indicates it is appropriate. Simply administering an antidepressant does not address the client's readiness to participate in a teaching session and ultimately self-care of the ostomy. The nurse should encourage the client to verbalize his feelings. Client education is not effective unless the client is receptive to the information. Readiness to learn is important. Proceeding with teaching when the client is struggling with coping is not sensitive to the client's individual needs. PTS:1DIF:ModerateREF:p. 120 KEY: Nursing process: Implementation | Client need: PHI | Cognitive level: Application

What makes a nursing history different from a medical history? 1) A nursing history focuses on the patient's responses to the health problem. 2) The same information is gathered; the difference is in who obtains the information. 3) A nursing history is gathered using a specific format. 4) A medical history collects more in-depth information.

ANS: 1 A medical history focuses on the patient's current and past medical/surgical problems. A nursing history focuses on the patient's responses to and perception of the illness/injury or health problem, his coping ability, and resources and support. Nursing history formats vary depending on the patient, the agency, and the patient's needs. Both nursing and medical histories typically use a specific format. A medical history does not necessarily contain more in-depth information. A nursing history can be in-depth, covering a wide range of topics, including biographical data, reason(s) patient is seeking healthcare, history of present illness, patient's perception of health status and expectations for care, past medical history, use of complementary modalities, and review of functional ability associated with activities of daily living. Other topics might deal with nutrition, psychosocial needs, pain assessment, or other special needs topics. PTS:1DIF:ModerateREF:pp. 44-45 KEY: Nursing process: Assessment | Client need: SECE | Cognitive level: Comprehension

How are standardized (model) care plans similar to unit standards of care? Standardized (model) care plans 1) Describe the care needed by patients in defined situations 2) Include specific goals and nursing orders 3) Become a part of the patient's comprehensive care plan 4) Usually describe ideal nursing care

ANS: 1 All of the statements are true for standardized care plans, but only 1 is true of both standardized care plans and unit standards of care. Both describe care needed by patients in defined situations, although unit standards usually describe care for groups of patients (e.g., all women admitted to a labor unit), and standardized care plans are often organized around a particular or all nursing diagnoses commonly occurring with a particular medical diagnosis. Unit standards are more general and do not have goals for each patient. Unit standards are kept on file in a central place on the unit and do not become a part of the care plan. Unit standards describe minimal, not ideal, care. PTS: 1 DIF: Difficult REF: p. 87; requires analysis of text discussion. KEY: Nursing process: Planning | Client need: SECE | Cognitive level: Analysis

What do initial, ongoing, and discharge planning have in common? 1) They are based on assessment and diagnosis. 2) They focus on the patient's perception of his needs. 3) They require input from a multidisciplinary team. 4) They have specific timelines in which to be completed.

ANS: 1 All planning is based on nursing assessment data and identified nursing diagnoses. The patient should have input, but the planning is based on the nursing assessment. The different types of planning are intertwined and may or may not be done at distinct, separate times. Discharge planning often requires a multidisciplinary team, but initial and ongoing planning may not. Initial planning is usually begun after the first patient contact, but there is no specified time for completion; ongoing planning is more or less continuous and is done as the need arises; discharge planning must be done before discharge. PTS:1DIF:ModerateREF: p. 81-82 KEY: Nursing process: Planning | Client need: SECE | Cognitive level: Analysis

What do the nursing assessment models have in common? 1) They assess and cluster data into model categories. 2) They organize assessment data according to body systems. 3) They specify use of the nursing process to collect data. 4) They are based on the ANA Standards of Care.

ANS: 1 All the models categorize or cluster data into functional health patterns, domains, or categories. None of the assessment models clusters data according to body system. Assessment is the first step of the nursing process; the nurse does not use the entire nursing process in data collection. The ANA Standards of Care describe a competent level of clinical nursing practice based on the nursing process; nursing models are not based on the ANA Standards of Care. PTS:1DIFgrinifficultREF:pp. 48

Which of the following is an example of an ongoing assessment? 1)Taking the patient's temperature 1 hour after giving acetaminophen (Tylenol) 2)Examining the patient's mouth at the time she complains of a sore throat 3)Requesting the patient to rate intensity on a pain scale with the first perception of pain 4)Asking the patient in detail how he will return to his normal exercise activities

ANS: 1 An ongoing assessment occurs when a previously identified problem is being reassessed—for example, taking an hourly temperature when a patient has a fever. Examining the mouth is a focused assessment to explore the patient's complaint of sore throat. Asking for a pain rating is a focused assessment at the first complaint of pain. A detailed interview about exercise is a special needs assessment; there is no way to know if it is initial or ongoing. PTS:1DIF:ModerateREF: p. 41

The diagnostic label, or patient problem, is used primarily to suggest 1) Client goals 2) Cue clusters 3) Interventions 4) Etiology

ANS: 1 As a general rule, the problem suggests goals for client outcomes. The etiology suggests interventions. Cue clusters support whether the correct nursing diagnosis has been identified. PTS: 1 DIF: Moderate REF: p. 73 KEY: Nursing process: Diagnosis | Client need: SECE | Cognitive level: Recall

The nurse has just been assigned to the clinical care of a newly admitted patient. To know how to best care for the patient, the nurse uses the nursing process. Which step would the nurse probably do first? 1)Assessment 2)Diagnosis 3)Plan outcomes 4)Plan interventions

ANS: 1 Assessment is the first step of the nursing process. The nursing diagnosis is derived from the data gathered during assessment, outcomes from the diagnosis, and interventions from the outcomes. PTS:1DIF:EasyREF: p. 30-31

The primary provider prescribes an indwelling urinary catheter for a client who is mildly confused and has been combative. How should the nurse proceed? 1) Ask a colleague for help, because the nurse cannot safely perform the procedure alone. 2) Gather the equipment and prepare it before informing the client about the procedure. 3) Obtain an order to restrain the client before inserting the urinary catheter. 4) Inform the provider that the nurse cannot perform the procedure because the client is confused.

ANS: 1 Before the nurse begins a procedure, she should review the care plan and look at the orders critically. Because this client is confused, she should ask a colleague to assist with the procedure to prevent undue stress for the client and nurse. The client should be informed about the procedure before the nurse gathers the equipment. Gathering the equipment and bringing it into the room before explaining the procedure might cause the client anxiety. Restraining the client should be done only as a last resort and to prevent client injury. Informing the primary provider that the procedure cannot be performed because the client is confused is inappropriate because the procedure can very likely be done with assistance. PTS:1DIF:ModerateREF:p. 118 KEY: Nursing process: Implementation | Client need: SECE | Cognitive level: Analysis

Which of the following is a criticism of standardized nursing diagnoses developed by NANDA-I? 1) There is little research to support nursing diagnoses labels. 2) A perfect nursing diagnosis must be written for it to be useful. 3) They are not included in all states' nurse practice acts. 4) Other professions do not recognize nursing diagnoses.

ANS: 1 Best practice is evidence-based practice; that is, it is developed through sound, scientific research. Research is currently being conducted, but many of the diagnoses are not research based. A perfect nursing diagnosis is impossible to write, so that is not an issue. Having standardized nursing diagnoses recognized in state practice acts or by other professions has nothing to do with the value of the NANDA-I taxonomy. PTS:1DIFgrinifficultREF: p. 57 KEY: Nursing process: Diagnosis | Client need: SECE | Cognitive level: Recall

How are NANDA-I problem labels and NOC outcome labels alike? Both describe 1) Health status in terms of human responses 2) Patient response before interventions are done 3) Patient response in positive terms 4) A pattern of related cues

ANS: 1 Both NANDA-I and NOC labels are stated as human responses. A NOC label can be used to describe patient responses both before and after intervention—NANDA-I before. NOC statements are neutral to allow for positive, negative, or no change in health status; NANDA-I diagnoses describe both problem responses and positive responses (wellness labels). NANDA-I labels are based on patterns of related cues; NOC labels are based on (linked to) NANDA-I labels. PTS: 1 DIF: Difficult REF: pp. 94; also information about NANDA-I diagnoses from Chapter 4 KEY: Nursing process: Planning | Client need: SECE | Cognitive level: Analysis

Which definition best describes a critical pathway? 1) Standardized plan of care for frequently occurring conditions 2) Systematically developed statement to assist practitioners and patients in making decisions 3) Systematic review of clinical evidence for an intervention 4) Set of interrelated concepts that describes or explains something

ANS: 1 Critical pathways are standardized plans of care for commonly occurring health conditions (e.g., myocardial infarction) for which similar outcomes and interventions are appropriate for the majority of patients with the condition. Clinical practice guidelines are systematically developed statements to assist practitioners and patients in making decisions about appropriate healthcare for a particular disease or procedure. Evidence reports are systematic reviews on clinical topics for the purpose of providing evidence for guidelines, quality improvement, quality measures, and insurance coverage decisions. A theory is a set of interrelated concepts that describe or explain something. PTS:1DIF:EasyREF: p. 104 KEY:Nursing process: N/A | Client need: SECE | Cognitive level: Recall

Which statement accurately describes delegation? 1) Transferring authority to another person to perform a task in a selected situation 2) Collaborating with other caregivers to make decisions and plan care 3) Scheduling treatments and activities with other departments 4) Performing a planned intervention from a critical pathway

ANS: 1 Delegation is the transfer to another person of the authority to perform a task in a selected situation—the person delegating retains accountability for the outcome of the activity. Collaboration is described as working with other caregivers to plan, make decisions, and perform interventions. Coordination of care involves scheduling treatments and activities with other departments. Implementation is the process of performing planned interventions. PTS:1DIF:EasyREF: p. 122 KEY: Nursing process: Implementation | Client need: SECE | Cognitive level: Knowledge

A client is admitted to the hospital with an acute respiratory problem resulting from lung disease. The nurse makes a diagnosis of Ineffective Breathing Pattern related to inability to maintain adequate rate and depth of respirations. Which nursing intervention should be listed first on the care plan? 1) Determine airway adequacy hourly and as needed. 2) Administer oxygen as needed. 3) Monitor arterial blood gas values. 4) Place the client in a high Fowler's position.

ANS: 1 For any acute respiratory problem, prior to implementing interventions, the nurse would assess breathing status of the patient by checking the respiratory rate and depth. When devising a plan of care for the client, nursing interventions should be listed according to priority. Airway always takes precedence, as ventilation, oxygenation, and positioning will be ineffective without a patent airway. PTS: 1 DIF: Difficult REF: p. 107 KEY: Nursing process: Planning | Client need: Physiological integrity | Cognitive level: Analysis

The nurse is interviewing a patient who has a recent onset of migraine headaches. The patient is highly anxious and cannot seem to focus on what the nurse is saying. Which of the following questions would be best for the nurse to use to begin gathering data about the headaches? 1) "When did your migraines begin?" 2) "Tell me about your family history of migraines." 3) "What are the types of things that trigger your headaches?" 4) "Describe what your headaches feel like."

ANS: 1 For someone who is anxious, it is best to use closed questions. (When did your migraines begin?) A closed question can be answered in one or very few words and has a very specific answer. The other questions are open-ended questions. PTS:1DIF:ModerateREF:p. 45 KEY: Nursing process: Assessment | Client need: PSI | Cognitive level: Application

In caring for a patient with comorbidities, the nurse draws upon her knowledge of diabetes and skin integrity. In a spirit of inquiry, she looks up the latest guidelines for providing skin care and includes them in the plan of care. The nurse provides skin care according to the procedural guidelines and begins regular monitoring to evaluate the effectiveness of the interventions. These activities are best described as 1)Full-spectrum nursing 2)Critical thinking 3)Nursing process 4)Nursing knowledge

ANS: 1 Full-spectrum nursing (1) involves the use of critical thinking, nursing knowledge, nursing process, and patient situation. Although the other answers are important for planning and delivering nursing care, they do not reflect all the nurse has demonstrated. PTS:1DIFgrinifficultREF:pp. 32-33; high-level question, answer not stated verbatim

Which type of client-centered evaluation is performed at specific, scheduled times? 1) Intermittent 2) Ongoing 3) Terminal 4) Process

ANS: 1 Intermittent evaluation is performed at specific times; it enables the nurse to judge the progress toward goal achievement and to modify the plan of care as needed. Ongoing evaluation is performed while implementing, immediately after an intervention, or with each client contact; these are not necessarily scheduled events. Terminal evaluation is performed at the time of discharge. It describes the client's health status and progress toward goals at that time. Process evaluation focuses on the manner in which care is given. It may be performed at specific times, but it is not considered a client-centered evaluation. PTS:1DIF:EasyREF: p. 127 KEY: Nursing process: Evaluation | Client need: SECE | Cognitive level: Recall

The nurse is individualizing Mr. Wu's plan of care by writing a plan for his nursing diagnosis of Anxiety. Why does the nurse need to write goals/outcomes on the plan of care? Because outcomes describe 1) Desired changes in the patient's health status 2) Specific patient responses to medical interventions 3) Specific nursing behaviors to improve a patient's health 4) Criteria to evaluate the appropriateness of a nursing diagnosis

ANS: 1 Outcomes describe changes in the patient's health status in response to nursing, rather than medical, interventions. Outcomes relate to patient behavior, not nursing behaviors. Outcomes are a measure of the effectiveness of nursing care for a specific nursing diagnosis, not whether the nursing diagnosis is appropriate. PTS:1DIF:ModerateREF: p. 91 KEY: Nursing process: Planning | Client need: SECE | Cognitive level: Comprehension

How are short-term goals different from long-term goals? Short-term goals 1) Can be met within a few hours or a few days 2) Are developed from the problem side of the nursing diagnosis 3) Must have target times/dates 4) Specify desired client responses to interventions

ANS: 1 Short-term goals may be accomplished in hours or days; long-term goals usually are achieved over weeks, months, or even years. The other statements are true for both short-term and long-term goals. PTS:1DIF:ModerateREF: p. 91 KEY: Nursing process: Planning | Client need: SECE | Cognitive level: Analysis

Which standardized intervention vocabulary was designed specifically for community health nurses? 1) Omaha System 2) Clinical Care Classification 3) Nursing Interventions Classification 4) International Classification for Nursing Practice

ANS: 1 The Omaha System was designed specifically for community health nurses to use in caring for individuals, families, community groups, or entire communities. The Clinical Care Classification was developed for home healthcare. The Nursing Interventions Classification system is applicable in all settings, including home health and community nursing. The International Classification for Nursing Practice system was designed to describe nursing practice of individuals, families, and communities worldwide. PTS:1DIF:EasyREF: p. 110 KEY: Nursing process: Planning | Client need: SASE | Cognitive level: Recall

A nurse admits a patient to the unit after completing a comprehensive interview and physical examination. To develop a nursing diagnosis, the nurse must now 1)Analyze the assessment data 2)Consult standards of care 3)Decide which interventions are appropriate 4)Ask the client's perceptions of her health problem

ANS: 1 The basis of the nursing diagnosis is the assessment data. Standards of care are referred to when establishing nursing interventions. Customizing interventions personalizes nursing care. Asking the patient about her perceptions is a method to validate whether the nurse has chosen the correct nursing diagnosis and would probably have been done during the comprehensive assessment. PTS: 1 DIF: Moderate REF: p. 31

Which of the following characteristics do the various definitions of critical thinking have in common? Critical thinking 1)Requires reasoned thought 2)Asks the questions "why?" or "how?" 3)Is a hierarchical process 4)Demands specialized thinking skills

ANS: 1 The definitions listed in the text as well as definitions in Box 2-1 state that critical thinking requires reasoning or reasoned thinking. Critical thinking is neither linear nor hierarchical. That means that the steps involved in critical thinking are not necessarily sequential, where mastery of one step is necessary to proceed to the next. Critical thinking is a purposeful, dynamic, analytic process that contributes to reasoned decisions and sound contextual judgments. PTS:1DIF:ModerateREF: p. 25; high-level question, answer not stated verbatim KEY: Nursing process: N/A | Client need: SECE | Cognitive level: Analysis

The patient verbalizes an overwhelming lack of energy. He says, "I still feel exhausted even after I sleep. I feel guilty when I can't keep up with my usual daily activities or sleep during the day. I've been a little depressed lately, too." The patient seems to have difficulty concentrating but has no apparent physical problems. Which of the following diagnoses best describes his health status? 1) Fatigue related to depression 2) Fatigue related to difficulty concentrating 3) Guilt related to lack of energy 4) Chronic confusion related to lack of energy

ANS: 1 The diagnosis that best describes the overall health status is Fatigue. The only cue that might cause Fatigue is depression. You cannot use depression as the problem because it is a medical diagnosis, and it is not a NANDA-I label. The other cues (difficulty concentrating, inability to perform ADLs, and guilt) are symptoms of Fatigue, not etiologies. These diagnoses would lead the nurse to focus on dealing with guilt and confusion, so the source of the Fatigue would not be addressed. PTS: 1 DIF: Difficult REF: pp. 70-73 KEY: Nursing process: Diagnosis | Client need: SECE | Cognitive level: Application

Which task can be delegated to nursing assistive personnel (NAP)? 1) Turn and reposition the client every 2 hours. 2) Assess the client's skin condition. 3) Change pressure ulcer dressings every shift. 4) Apply hydrocolloid dressing to the pressure ulcer.

ANS: 1 The nurse can delegate turning the client every 2 hours to the NAP. Assessing the client's skin condition, changing pressure ulcer dressings, and applying a hydrocolloid dressing to a pressure ulcer are all interventions that require nursing knowledge and judgment. PTS:1DIF:ModerateREF:pp. 122-124 KEY: Nursing process: Implementation | Client need: SECE | Cognitive level: Application

The nurse is using electronic care planning. He enters the patient's nursing diagnosis into the computer and chooses desired outcomes. He has validated his data, diagnosis, and goals. When he considers the list of interventions the program generates, he sees that none of them fit this patient's individual needs. What should the nurse do? 1) Reject them all and type in appropriate interventions. 2) Select the interventions from the program that are most suitable. 3) Ask another nurse to assess the patient and give her recommendation. 4) Restart the computer; it is probably a program malfunction.

ANS: 1 The nurse can reject all the suggested interventions if they do not address patient needs. Nearly all computer programs have a screen that allows you to type in interventions and nursing orders. It is the nurse's responsibility to choose interventions: He cannot abdicate this responsibility and let the computer "choose." As a professional, this nurse has already validated the data, nursing diagnosis, and goals, so he can feel reasonably certain that there is nothing wrong with the plan to that point. Although consultation with other nurses can be wise and prudent at times, the nurse caring for the patient would likely have the most familiarity with the healthcare needs and is in a better position to make sound judgments than another nurse who does not know the patient. Therefore, it might not be productive or efficient to consult another nurse or restart the computer. PTS:1DIF:ModerateREF:p. 108-109 KEY: Nursing process: Planning | Client need: SECE | Cognitive level: Analysis

Who is responsible for evaluating the outcome of a task delegated to the nursing assistive personnel (NAP)? 1) Nurse who delegated the task 2) Licensed practical nurse working with the NAP 3) Unit nurse manager 4) Charge nurse for the shift

ANS: 1 The nurse who delegates the task is responsible for supervising and evaluating the outcomes of tasks performed by the NAP. Another registered nurse, such as a staff nurse, nurse manager, or charge nurse, can answer questions and provide help, if necessary. PTS:1DIF:EasyREF: p. 124 KEY: Nursing process: Implementation | Client need: SECE | Cognitive level: Recall

A 55-year-old patient returned to the medical-surgical unit after undergoing a right hemicolectomy (abdominal surgery) for colon cancer. Which of the following is an appropriate, correctly written nursing order for this patient? 1) 7/12/13 Encourage use of the incentive spirometer every hour while the client is awake—D. Goodman, RN 2) By 7/12/13, uses incentive spirometer 10 times every hour while awake to 1000 mL 3) Incentive spirometer hourly while awake 4) Offer incentive spirometer to the client—J. Smith, RN

ANS: 1 The option beginning with a date and ending with the RN's signature contains necessary information. It contains the date the order was written along with specific instruction for the nurse that is written in terms of nursing behavior. "Uses incentive spirometer 10 times . . ." is an example of an expected outcome. "Incentive spirometer hourly . . ." is an example of a medical order. Plus, the date and nurse's signature are missing. "Offer incentive spirometer . . ." does not provide the nurse with enough detailed instruction. Therefore, it is a poorly written nursing order. PTS:1DIF:ModerateREF:pp. 111-112 KEY: Nursing process: Planning | Client need: Physiological integrity | Cognitive level: Application

Which of the following nursing diagnoses is written in correct format? Assume the facts are correct in all of them. 1) Readiness for Enhanced Nutrition 2) Pain related to stating, "On a scale of 1 to 5, it's a 5." 3) Impaired Mobility related to pain A.M.B. hip fracture 4) Risk for Infection related to compromised immunity A.M.B. fever

ANS: 1 Wellness diagnoses (e.g., Readiness for Enhanced . . .) are usually one-part statements. A pain ranking of 5 is a symptom of pain, not an etiology, so it should be preceded by "A.M.B." or "as manifested by." Hip fracture is a medical diagnosis that is causing an etiology of pain; therefore, it should be preceded by "secondary to." Risk diagnoses do not have symptoms, so it is not correct to put anything after "A.M.B." PTS: 1 DIF: Moderate REF: pp. 70-73 KEY: Nursing process: Diagnosis | Client need: SECE | Cognitive level: Application

Which of the following questions would be effective for obtaining information from a patient? Choose all that apply. 1) "How did this happen to you?" 2) "What was your first symptom?" 3) "Why didn't you seek healthcare earlier?" 4) "When did you start having symptoms?"

ANS: 1, 2, 4 How, what, and when are acceptable lines of questioning. Asking "why" can put the patient on the defensive and may suggest disapproval, limiting the amount of information the patient is willing give. PTS:1DIF:ModerateREF:p. 45-46 KEY: Nursing process: Assessment | Client need: SECE | Cognitive level: Application

Which aspects of healthcare are affected by a client's culture? Select all that apply. 1)How the clients views healthcare 2)How the client views illness 3)How the client will pay for healthcare services 4)The types of treatments the client will accept 5)When the client will seek healthcare services 6)The environment where the healthcare services are provided 7)The ease of accessibility of healthcare services

ANS: 1, 2, 4, 5 Culture affects clients' view of health and healthcare. It influences how they will define illness, when they will seek healthcare, and what treatments are acceptable in their culture. How services are paid for is related to economic status. Regardless of culture, anyone can be affected by previous healthcare experiences, the environment in which healthcare is provided, and accessibility of services. PTS:1DIF:ModerateREF:p. 27 KEY:Nursing process: N/A | Client need: SECE | Cognitive level: Recall

Which statement(s) about nursing interventions is/are true? Select all that apply. 1) The responsibility of writing nursing orders cannot be delegated to the LPN/LVN. 2) The best nursing interventions are based on tradition. 3) Nursing interventions should be individualized and culturally sensitive. 4) Standardized nursing interventions improve care for a specific client.

ANS: 1, 3 Some nursing interventions and activities can be delegated to the LPN/LVN or nursing assistive personnel (NAP); however, writing nursing orders is the responsibility of the registered nurse. Nursing interventions should always be individualized and culturally sensitive. Whenever possible, nursing interventions should be based on scientific evidence, not tradition. Standardized interventions are not customized to improve care for a specific client. PTS:1DIF:ModerateREF: pp. 103-104 KEY: Nursing process: Interventions | Client need: SASE | Cognitive level: Application

Which of the following are cues rather than inferences? Choose all correct answers. 1) Ate 50% of his meal 2) Patient feels better today 3) States, "I slept well" 4) White blood cell count 15,000/mm3

ANS: 1, 3, 4 Cues are what the client says and what you observe. "Just the facts." The only inference in the list is "slept well." What did the nurse observe to tell her the client slept well? Those would be cues. If the client states, "I slept well" it is a cue, because it is a fact—that is what the client stated. PTS:1DIF:EasyREF:p. 50 KEY: Nursing process: Assessment | Client need: SECE | Cognitive level: Comprehension

An 80-year-old resident in a long-term-care facility comes to the emergency department with dehydration. The nurse writes a diagnosis of Deficient Fluid Volume related to excessive fluid loss. An individualized nursing goal identified for this client is "The client will maintain urine output of at least 30 mL/hour." Which nursing interventions would directly help achieve or evaluate the stated goal? Choose all that are correct. 1) Measure and record urine output every hour; report an output of less than 30 mL/hour. 2) Monitor skin turgor and moistness of mucous membranes every shift. 3) Administer IV fluids as prescribed. 4) Keep oral fluids within the patient's reach, and encourage the patient to drink.

ANS: 1, 3, 4 Measuring and recording urine output allow for direct evaluation of the goal "urine output 30 mL/hour." Administering IV fluids adds fluid to correct dehydration, improve blood flow through the kidneys, and increase urine production. Intake of oral fluids has the same effect. Monitoring skin turgor and mucous membranes are ways to assess for dehydration, but it does not directly apply to the goal of maintaining urine output. It is an intervention aimed at the etiology of this nursing diagnosis, rather than the problem. PTS:1DIF:ModerateREF:p. 106-107; high-level question; answer not given verbatim KEY: Nursing process: Interventions | Client need: Physiological integrity | Cognitive level: Analysis

The nurse and nursing assistive personnel (NAP) are caring for a group of patients on the medical-surgical floor. For which of the following patients can the nurse delegate to the NAP the task of bathing? Choose all that apply. 1) 75-year-old patient newly admitted to the hospital with dehydration 2) 65-year-old patient hospitalized for a stroke, whose blood pressure is 188/90 mm Hg 3) 92-year-old patient with stable vital signs who was admitted with a urinary tract infection 4) 56-year-old patient with chronic renal failure who has vital signs within his normal range

ANS: 1, 3, 4 The nurse should not delegate bathing of a client newly diagnosed with a stroke whose blood pressure is unstable or otherwise abnormal. This client requires the keen assessment and critical thinking skills of a registered nurse. The nurse can safely delegate the care of stable clients, such as the client admitted with dehydration, the client admitted with a urinary tract infection, or the client with chronic renal failure. Any client who is very ill or who requires complex decision making should be cared for by a registered nurse. PTS: 1 DIF: Difficult REF: pp. 122-124 KEY: Nursing process: Implementation | Client need: SECE | Cognitive level: Analysis

Which of the following is an example of a problem that nurses can treat independently? 1) Hemorrhage 2) Nausea 3) Fracture 4) Infection

ANS: 2 A nursing diagnosis (or nursing problem) is a human response to a disease, injury, or other stressor that nurses can identify, prevent, or treat independently. Nausea is the only problem that meets that criterion; all others are medical or collaborative problems. PTS:1DIF:ModerateREF: pp. 57 KEY: Nursing process: Diagnosis | Client need: PHSI | Cognitive level: Application

A nurse is caring for an 80-year-old patient of Chinese heritage. When planning outcomes for this patient, which actions by the nurse would meet the American Nurses Association standards for outcomes identification? Choose all that apply. 1) Developing culturally appropriate outcomes 2) Using the outcomes preprinted on the clinical pathway 3) Choosing the best outcome for the patient, regardless of the costs involved in bringing it about 4) Involving the patient and family in formulating the outcomes

ANS: 1, 4 ANA standard 3 includes "derives culturally appropriate expected outcomes from the diagnosis" and "involves the patient, family . . . in formulating expected outcomes. . . ." It is acceptable for the nurse to use outcomes on a clinical pathway, but these are not individualized; ANA standard 3 says that the nurse "identifies . . . outcomes for a plan individualized to the patient. . . ." The standard also says that the nurse should consider "associated risks, benefits, and costs. . . ." PTS:1DIF:ModerateREF: p. 82 KEY: Nursing process: Planning | Client need: SECE | Cognitive level: Comprehension

A nurse with a large caseload of patients needs to delegate some assessment tasks to other members of the health team. The nurse is unsure which tasks can be delegated to nursing assistive personnel (NAP) and which are appropriate for a licensed practical nurse (LPN) or a registered nurse (RN). To which sources should the nurse turn for the answer to his question? Choose all that are appropriate. 1) The nurse practice act of his state 2) The American Medical Association guidelines 3) The Code of Ethics for Nurses 4) The American Nurses Association's Scope and Standards of Practice

ANS: 1, 4 State nurse practice acts specify which portions of the assessment can legally be completed by individuals with different credentials. The ANA Scope and Standards of Practice provide a guide for who is ultimately responsible and qualified to collect assessment data. The American Medical Association provides guidelines and standards for physicians, not nurses. The Code of Ethics for Nurses says merely that the nurse should delegate tasks appropriately; it does not speak to credentials of personnel. PTS:1DIF:ModerateREF:p. 40 KEY: Nursing process: Assessment | Client need: SECE | Cognitive level: Comprehension

Which of the following is a client outcome criterion? 1) Central venous catheter site infection does not occur (90% of cases). 2) Client will sit out of bed in a chair for 20 minutes three times per day. 3) Postoperative phlebitis does not occur (95% of cases). 4) Falls will decrease by 2% between January 1 and March 30.

ANS: 2 A client outcome criterion states the client health status or behaviors one wishes to effect. "Client will sit out of bed . . ." is a client outcome criterion. The other options are examples of organizational criteria used to evaluate the quality of care throughout the institution. PTS:1DIF:ModerateREF: pp. 127-128 KEY: Nursing process: Evaluation | Client need: SECE | Cognitive level: Application

For which patient would it be most important to perform a comprehensive discharge plan? 1) A teen who is a first-time mother, single, and lives with her parents 2) An older adult who has had a stroke affecting the left side of his body and lives alone 3) A middle-aged man who has had outpatient surgery on his knee and requires crutches 4) A young woman who was admitted to the hospital for observation following an accident

ANS: 2 A comprehensive discharge plan should be developed for older adults and anyone who has complex needs, including self-care deficits. The other patients do not have the complex needs of the older adult patient who has had a stroke that affects body function. PTS:1DIF:ModerateREF:p. 83 KEY: Nursing process: Planning | Client need: PHSI | Cognitive level: Analysis

Which of the following is the most valid criterion for determining the status of a patient's anxiety at discharge? The patient 1) Has a relaxed facial expression 2) States that he feels more relaxed today 3) Shows no physiological signs of anxiety (e.g., pallor) 4) Has no further questions about home care

ANS: 2 A criterion is considered valid when it measures what it is intended to measure. Because anxiety is subjective (perceived by the patient), the best measure of anxiety is what the patient says about it. A relaxed facial expression and other physiological signs might or might not show the level of anxiety. Relaxation might occur, for example, because the patient is sleeping or falling asleep. The fact that a patient is not asking questions about his surgery could mean that he has adequate knowledge about the topic; it would not indicate the presence or absence of anxiety. All of the options except what the patient states could be measuring something other than anxiety. PTS: 1 DIF: Difficult REF: p. 127 KEY: Nursing process: Evaluation | Client need: PSI | Cognitive level: Application

A few nurses on a unit have proposed to the nurse manager that the process for documenting care on the unit be changed. They have described a completely new system. Why is it important for the nurse manager to have a critical attitude? It will help the manager to 1)Consider all the possible advantages and disadvantages 2)Maintain an open mind about the proposed change 3)Apply the nursing process to the situation 4)Make a decision based on past experience with documentation

ANS: 2 A critical attitude enables the person to think fairly and keep an open mind. PTS:1DIF:ModerateREF:pp. 26 KEY: Nursing process: N/A | Client need: SECE | Cognitive level: Comprehension

The most obvious reason for using a framework when assessing a patient is to 1) Prioritize assessment data 2) Organize and cluster data 3) Separate subjective and objective data 4) Identify primary from secondary data

ANS: 2 A framework is used to organize and cluster data to find patterns. During the assessment phase, the nurse is collecting and recording data, not prioritizing the data. A framework includes subjective and objective data as well as primary and secondary data; it does not help you to separate them. PTS:1DIF:EasyREF:p. 48 KEY: Nursing process: Assessment | Client need: SECE | Cognitive level: Recall

A patient has left-sided weakness because of a recent stroke. Which type of special needs assessment would it be most important to perform? 1) Family 2) Functional 3) Community 4) Psychosocial

ANS: 2 A functional assessment is most important because of discharge needs (e.g., self-care ability at home) and patient safety. A family and community assessment would be helpful to evaluate support systems, and a psychosocial assessment would be helpful to evaluate a patient's understanding of and coping with his recent stroke. Remember that special needs assessments are lengthy and time-consuming, so they should be used only when in-depth information is needed about a topic. PTS:1DIF:ModerateREF:pp. 43 KEY: Nursing process: Assessment | Client need: SECE | Cognitive level: Analysis

How does a risk nursing diagnosis differ from a possible nursing diagnosis? 1) A risk diagnosis is based on data about the patient. 2) A possible diagnosis is based on partial (or incomplete) data. 3) Nurses collect the data to support risk diagnoses. 4) A possible diagnosis becomes an actual diagnosis when symptoms develop.

ANS: 2 A possible nursing diagnosis is based on nursing knowledge, intuition, and experience and does not have enough data to support it; it is based on incomplete data. A risk diagnosis describes a problem that may develop in a vulnerable client if nursing care is not initiated to prevent it; it is made when risk factors are present in the data. Nurses collect data to support both risk and possible diagnoses; therefore, this statement does not differentiate them. A risk diagnosis becomes an actual diagnosis when symptoms develop. PTS:1DIF:ModerateREF:p. 60 KEY: Nursing process: Diagnosis | Client need: SECE | Cognitive level: Analysis

For which of the following purposes is a graphic flow sheet superior to other methods of recording data? 1) Easy documentation of routine vital signs 2) Seeing the patterns of a patient's fever 3) Describing the symptoms accompanying a rising temperature 4) Checking to make sure vital signs were taken

ANS: 2 All are benefits of the graphic flow sheet, but to easily and graphically see trends over time, the graphic flow sheet is superior to other methods of documentation. For the other options, other kinds of flow sheets would be equally effective. PTS:1DIF:ModerateREF:p. 50 KEY: Nursing process: N/A | Client need: SECE | Cognitive level: Analysis

The nurse works with the respiratory therapist to administer a patient's breathing treatments. He reports the patient's breathing status and tolerance of the treatment to the primary care provider. The nurse then discusses with the patient the options for further treatment. This is an example of 1) Delegation 2) Collaboration 3) Coordination of care 4) Supervision of care

ANS: 2 Collaboration means working with other caregivers to plan, make decisions, and perform interventions. Delegation is the transfer to another person of the authority to perform a task in a selected situation. Coordination of care involves scheduling treatments and activities with other departments, putting together all the patient data to obtain the "big picture." Supervision is the process of directing, guiding, and influencing the outcome of an individual's performance of an activity or task. PTS:1DIF:ModerateREF: pp. 122 KEY: Nursing process: Evaluation | Client need: SECE | Cognitive level: Application

What do critical thinking and the nursing process have in common? 1)They are both linear processes used to guide one's thinking. 2)They are both thinking methods used to solve a problem. 3)They both use specific steps to solve a problem. 4)They both use similar steps to solve a problem.

ANS: 2 Critical thinking and the nursing process are ways of thinking that can be used in problem solving (although critical thinking can be used beyond problem-solving applications). Neither method of thinking is linear. The nursing process has specific steps; critical thinking does not. PTS:1DIFgrinifficultREF: p. 31

Which of the following examples includes both objective and subjective data? 1)The client's blood pressure is 132/68 and her heart rate is 88. 2)The client's cholesterol is elevated, and he states he likes fried food. 3)The client states she has trouble sleeping and that she drinks coffee in the evening. 4)The client states he gets frequent headaches and that he takes aspirin for the pain.

ANS: 2 Elevated cholesterol is objective, and "states he likes fried food" is subjective. Objective data can be observed by someone other than the patient (e.g., from physical assessments or lab and diagnostic tests). Subjective data are information given by the client. Blood pressure and heart rate measurements are both objective. "States . . . trouble sleeping and . . . drinks coffee . . ." are both subjective. States ". . . frequent headaches and . . . takes aspirin . . ." are both subjective. PTS:1DIF:ModerateREF:pp. 40 KEY: Nursing process: Assessment | Client need: PHSI | Cognitive level: Analysis

Which nursing intervention is considered an independent intervention? 1) Administering 1 L of dextrose 5% in normal saline solution at 100 mL/hour 2) Encouraging the postoperative client to perform coughing and deep breathing exercises 3) Explaining his diet to the client; then communicating the teaching with the dietitian 4) Administering morphine sulfate 2 mg IV to the client with postoperative pain

ANS: 2 Encouraging the postoperative client to perform coughing and deep breathing exercises is an independent nursing intervention. An independent intervention is one that nurses are licensed to prescribe, perform, or delegate based on their skills and knowledge. Administering IV fluid or morphine sulfate are dependent interventions; they require an order from a physician or advanced practice nurse but are carried out by the nurse. Explaining to the client how sodium intake affects his heart failure and then communicating the teaching with the dietitian is an interdependent intervention, one that is carried out in collaboration with other healthcare team members. PTS:1DIF:ModerateREF: p. 103 KEY: Nursing process: Interventions | Client need: SASE | Cognitive level: Application

Based only on Maslow's hierarchy of needs, which nursing diagnosis should have the highest priority? 1) Self-care Deficit 2) Risk for Aspiration 3) Impaired Physical Mobility 4) Disturbed Sensory Perception

ANS: 2 Highest priority is given to problems that are life threatening or that could be destructive to the client. Safety is most basic in Maslow's hierarchy. Even though Risk for Aspiration is not an actual problem, it poses the most immediate life-threatening risk to the client, and nursing interventions must be performed to prevent it from becoming an actual problem. PTS:1DIF:ModerateREF:pp. 64-65 KEY: Nursing process: Diagnosis | Client need: PHSI | Cognitive level: Analysis

What is wrong with the format of this diagnostic statement: Possible Risk for Constipation related to irregular defecation habits A.M.B. statement that "When I'm busy, I can't always take the time to go to the bathroom." 1) Possible nursing diagnoses do not have signs and symptoms. 2) A nursing diagnosis is either a possible risk or a risk, not both. 3) Constipation is a medical diagnosis. 4) The etiology is actually a defining characteristic.

ANS: 2 If there are risk factors, it is not a possible diagnosis, it is a risk diagnosis. It is possible to have a "possible risk for" diagnosis. The patient with possible diagnoses may have symptoms, just not enough to support the diagnosis. Constipation is a nursing diagnosis, and the etiology is a defining characteristic for a risk diagnosis because it contributes to the problem. In risk diagnoses, the etiology consists of the risk factors. PTS: 1 DIF: Moderate REF: pp. 60 KEY: Nursing process: Diagnosis | Client need: SECE | Cognitive level: Analysis

A client arrives in the emergency department, pale and breathing rapidly. He immediately becomes unconscious and collapses to the floor. The nurse rapidly assesses the patient and decides the first series of actions that are needed. This scenario demonstrates 1) Formal planning 2) Informal planning 3) Ongoing planning 4) Initial planning

ANS: 2 Informal planning is performed while doing other nursing process steps and is not written; this nurse is forming a plan in her mind. The end product of formal planning is a holistic plan of care that addresses the patient's unique problems and strengths; this nurse has no time to create a holistic plan of care. Ongoing planning refers to changes made in the plan as you evaluate the patient's responses to care; no care has been given at this point. Initial planning does indeed begin with the first patient contact. However, it refers to the development of the initial comprehensive plan or care; this nurse does not have enough data for a comprehensive plan, nor does she have time to make such a plan at the moment. PTS:1DIF:EasyREF:p. 81 KEY: Nursing process: Planning | Client need: SECE | Cognitive level: Analysis

How is NOC different from the Omaha System? 1) NOC can be used to write health restoration outcomes. 2) NOC can be used in all specialty and practice areas. 3) NOC can be used for individuals, families, or groups. 4) NOC formulates goals based on nursing diagnoses.

ANS: 2 NOC was developed for all specialty and practice areas. The Omaha System was developed for community health nursing. Both address health restoration and can be used for individuals, family, or groups (community). Both base goals on nursing diagnoses, although Omaha does not use the NANDA-I taxonomy. PTS:1DIF:ModerateREF: p. 95; answer based on analysis of text discussion | V1, p. 98; answer based on analysis of text discussion KEY: Nursing process: Planning | Client need: SECE | Cognitive level: Analysis

Before inserting a nasogastric tube, the nurse reassures the client. Reassuring the client requires which type of nursing skill? 1) Psychomotor 2) Interpersonal 3) Cognitive 4) Critical thinking

ANS: 2 Reassuring the client is an interpersonal skill. Inserting the nasogastric tube requires psychomotor skills. Checking catheter placement after insertion requires cognitive and psychomotor skills. Assessing whether there is an indication for the nasogastric tube requires critical thinking skills. PTS:1DIF:ModerateREF: p. 120 KEY: Nursing process: Implementation | Client need: PHI | Cognitive level: Comprehension

The nurse is preparing to admit a patient from the emergency department. The transferring nurse reports that the patient is obese. The nurse has been overweight at one time and works very hard now to maintain a healthy weight. She immediately thinks, "I know I tend to feel negatively about obese people; I figure if I can stop eating, they should be able to. I must remember how very difficult that is and be very careful not to be judgmental of this patient." This best illustrates 1)Theoretical knowledge 2)Self-knowledge 3)Using reliable resources 4)Use of the nursing process

ANS: 2 Self-knowledge is self-understanding—awareness of one's beliefs, values, biases, and so on. That best describes the nurse's awareness that her bias can affect her patient care. Theoretical knowledge consists of information, facts, principles, and theories in nursing and related disciplines; it consists of research findings and rationally constructed explanations of phenomena. Using reliable resources is a critical thinking skill. The nursing process is a problem-solving process consisting of the steps of assessing, diagnosing, planning outcomes, planning interventions, implementing, and evaluating. The nurse has not yet met this patient, so she could not have begun the nursing process. PTS:1DIFgrinifficultREF:pp. 30; high-level question, answer not stated verbatim

The Joint Commission requires which type of assessment to be performed on all patients? 1)Functional ability 2)Pain 3)Cultural 4)Wellness

ANS: 2 The Joint Commission requires that pain and nutrition assessment be performed on all patients. Other special needs assessments should be performed when cues indicate there are risk factors. PTS:1DIF:ModerateREF:p. 39 KEY: Nursing process: Assessment | Client need: PHSI | Cognitive level: Analysis

The nursing diagnosis is Impaired Memory related to fluid and electrolyte imbalances A.M.B. inability to recall recent events. Which of the following goals/outcomes must be included on the care plan? 1) Checks current medications for mind-altering side effects 2) Demonstrates use of techniques to help with memory loss 3) Drinks at least 1500 cc of fluid per day 4) Takes electrolyte supplements with meals

ANS: 2 The essential goal/outcome is aimed at the problem response Impaired Memory. The other goals in this question address the etiology. PTS:1DIF:ModerateREF: p. 93-94 KEY: Nursing process: Planning | Client need: PHSI | Cognitive level: Application

The nurse developed a care plan for a patient to help prevent Impaired Skin Integrity. She has made sure that nursing assistive personnel change the patient's position every 2 hours. In the evaluation phase of the nursing process, which of the following would the nurse do first? 1)Determine whether she has gathered enough assessment data. 2)Judge whether the interventions achieved the stated outcomes. 3)Follow up to verify that care for the nursing diagnosis was given. 4)Decide whether the nursing diagnosis was accurate for the patient's condition.

ANS: 2 The evaluation phase judges whether the interventions were effective in achieving the desired outcomes and helped to alleviate the nursing diagnosis. This must be done before examining the nursing process steps and revising the care plan. PTS:1DIF:ModerateREF: p. 31

Which of the following is an example of the most basic motivation in Maslow's hierarchy of needs? 1)Experiencing loving relationships 2)Having adequate housing 3)Receiving education 4)Living in a crime-free neighborhood

ANS: 2 The most basic needs are centered on physiological survival—shelter (housing), food, and water. All other options are for higher needs. The order from most basic to highest level is physiologic, safety and security, love and belonging, esteem, and self-actualization. Loving relationships fall under the love and belonging category. Education is a form of self-actualization. Living in a crime-free neighborhood meets the need for safety and security. PTS:1DIF:ModerateREF:p. 48 KEY: Nursing process: Assessment | Client need: PHSI | Cognitive level: Application

Which statement about the nursing process is correct? 1)It was developed from the ANA Standards of Care. 2)It is a problem-solving method to guide nursing activities. 3)It is a linear process with separate, distinct steps. 4)It involves care that only the nurse will give.

ANS: 2 The nursing process is a problem-solving process that guides nursing actions. The ANA organizes its Standards of Care around the nursing process, but the process was not developed from the standards. The nursing process is cyclical and involves care the nurses give or delegate to other members of the healthcare team. PTS:1DIF:EasyREF: p. 31

A client is admitted to the hospital with an acute episode of chronic obstructive pulmonary (lung) disease. The nurse makes a diagnosis of Ineffective Breathing Pattern related to inability to maintain adequate rate and depth of respirations and has recorded the diagnosis and appropriate goals on the care plan. When selecting nursing interventions, what should the nurse do first? 1) Identify several interventions likely to achieve the desired outcomes. 2) Review the problem and etiology of the nursing diagnosis. 3) Choose the best interventions for the patient. 4) Review the goals she has written.

ANS: 2 The process of choosing interventions is review the nursing diagnosis, review the desired outcomes, identify several interventions or actions, choose the best interventions for the patient, and then individualize standardized interventions to meet the patient's unique needs. PTS:1DIF:ModerateREF: p. 106-107 KEY: Nursing process: Planning | Client need: SECE | Cognitive level: Analysis

Which of the following is an example of theoretical knowledge? 1)A nurse uses sterile technique to catheterize a patient. 2)Room air has an oxygen concentration of 21%. 3)Glucose monitoring machines should be calibrated daily. 4)An irregular apical heart rate should be compared with the radial pulse.

ANS: 2 Theoretical knowledge consists of research findings, facts, principles, and theories. The oxygen concentration of room air is a scientific fact. The others are examples of practical knowledge—what to do and how to do it. PTS:1DIF:ModerateREF:p. 30; high-level question, answer not stated verbatim

Which of the following are cues? Select all that apply. 1) Taking a brisk walk five times a week 2) Using laxatives to have a bowel movement 3) Needing more sleep than usual 4) Decreasing the amount of fat in the diet 5) Weighing less than indicated by developmental norms

ANS: 2, 3, 5 Cues are a deviation from norms, such as changes in usual health behavior, indications of delayed growth and development, changes in behaviors, or nonproductive or dysfunctional behavior. PTS:1DIF:ModerateREF:p. 61-62 KEY: Nursing process: Diagnosis | Client need: SECE | Cognitive level: Application

Which of the following is an example of an active listening behavior? 1) Taking frequent notes 2) Asking for more details 3) Leaning toward the patient 4) Sitting with legs crossed

ANS: 3 Active listening behaviors include leaning toward the patient; facing the patient; open, relaxed posture without crossing arms or legs; and maintaining eye contact. Taking frequent notes makes it difficult to keep eye contact. Asking for more details may seem like idle curiosity. Sitting with legs crossed may indicate to the patient that you are not open to her. PTS:1DIF:EasyREF:p. 47 KEY: Nursing process: Assessment | Client need: SECE | Cognitive level: Comprehension

The patient shows the necessary defining characteristics, and the nurse has diagnosed Decisional Conflict related to unclear personal values and beliefs. What essential action should the nurse take to help ensure the accuracy of this diagnosis? 1) Ask a more experienced nurse to confirm it. 2) Have a social worker interview the patient. 3) Ask the patient to confirm the diagnosis. 4) Read about Decisional Conflict in the NANDA-I handbook.

ANS: 3 After identifying problems and etiologies (which this nurse has done), the nurse should verify them with the patient to help ensure that her conclusions are accurate. If the patient does not agree that he has Decisional Conflict, the nurse might interview him more to clarify the meaning of the data. Certainly the nurse could ask another nurse's opinion, but that is not essential. It would make no sense to have a social worker interview the patient unless the situation remains unclear even after confirming with the client. If the nurse did have adequate theoretical knowledge of Decisional Conflict for this patient, she should have been informed by reading the NANDA-I handbook before making the diagnosis. If the patient does not confirm the diagnosis, and the nurse concludes the diagnosis is in error, she might then reread the NANDA-I guide. PTS: 1 DIF: Moderate REF: V1, p. 56 | p. 63 KEY: Nursing process: Diagnosis | Client need: SECE | Cognitive level: Application

Which of the following is the best approach to validate a clinical inference? 1) Have another nurse evaluate it. 2) Have the physician evaluate it. 3) Have sufficient supportive data. 4) Have the client's family confirm it.

ANS: 3 All clinical inferences should be well supported by data. The more reliable data you gather, the more certain you can be that your inference is accurate. Because inferences are nursing diagnoses, it would be inappropriate to have a physician evaluate them. Although another experienced nurse could evaluate the inference, it still needs to be supported by sound and sufficient data. Even clients can validate clinical inferences in some situations, but adequate supporting data are still needed. Keep in mind that the client's data might or might not be sufficient to "prove" the inference. PTS: 1 DIF: Easy REF: p. 63 KEY: Nursing process: Diagnosis | Client need: SECE | Cognitive level: Recall

Which of the following nursing interventions is an indirect-care intervention? 1) Emotional support 2) Teaching 3) Consulting 4) Physical care

ANS: 3 An indirect-care intervention is an activity performed away from the client on behalf of the client. Indirect-care interventions include consulting with other healthcare team members, making referrals, advocacy, and managing the environment. Direct-care interventions include emotional support, patient teaching, and physical care. PTS:1DIF:EasyREF:pp. 103 KEY: Nursing process: Implementation | Client need: SASE | Cognitive level: Recall

A patient comes to the urgent care clinic because he stepped on a rusty nail. What type of assessment would the nurse perform? 1) Comprehensive 2) Ongoing 3) Initial focused 4) Special needs

ANS: 3 An initial focused assessment is performed during a first exam for specific abnormal findings. A comprehensive assessment is holistic and is usually done upon admission to a healthcare facility. An ongoing assessment follows up after an initial database is completed or a problem is identified. A special needs assessment is performed when there are cues that more in-depth assessment is needed. PTS:1DIF:ModerateREF: pp. 42-43 KEY: Nursing process: Assessment | Client need: PHSI | Cognitive level: Application

Which of the following is true for goals/outcomes for collaborative problems? 1) They are monitored only by other disciplines. 2) They are usually sensitive to nursing interventions. 3) They state that a complication will not occur. 4) They state only broad performance criteria.

ANS: 3 The goal for a collaborative problem is always that the complication will not occur. Other disciplines may be involved in helping to prevent the problem, but nurses still monitor for the complication. The outcomes to collaborative problems are not affected by nursing interventions alone. Goals for collaborative problems are specific to the medical condition/treatment. PTS:1DIF:ModerateREF: pp. 93 KEY: Nursing process: Planning | Client need: SECE | Cognitive level: Analysis

How are critical pathways and standardized nursing care plans similar? Both 1) Specify daily, or even hourly, outcomes and interventions 2) Prescribe minimal care needed to meet recommended lengths of stay 3) Describe care common to all patients with a certain condition or situation 4) Emphasize medical problems and interventions

ANS: 3 Both critical pathways and standardized care plans are preplanned documents; they describe care common to all patients who have a certain condition (e.g., all patients who have a heart attack need some of the same interventions). The other statements are true of critical pathways but not of standardized nursing care plans. PTS:1DIFgrinifficultREF: pp. 86-87; high-level question, answer not given verbatim KEY: Nursing process: Planning | Client need: SECE | Cognitive level: Analysis

Which statement by the nurse best demonstrates clear communication to nursing assistive personnel (NAP) about delegating a task? 1) "Record how much the patient drinks today, please." 2) "Take the patient's vital signs every 2 hours today." 3) "Take the patient's temperature every 4 hours; notify me if it is greater than 100.5°F (38.1°C)." 4) "Assist the patient with all of her meals."

ANS: 3 Clear communication about a task (such as "Take the patient's temperature . . . ") tells the NAP exactly what the task is, the specific time it needs to be done, and the method for reporting the results to the registered nurse. The other options are vague and leave room for misinterpretation. PTS:1DIF:ModerateREF:p. 124 KEY: Nursing process: Implementation | Client need: SECE | Cognitive level: Analysis

Which of the following is the most important reason for nurses to be critical thinkers? 1)Nurses need to follow policies and procedures. 2)Nurses work with other healthcare team members. 3)Nurses care for clients who have multiple health problems. 4)Nurses have to be flexible and work variable schedules.

ANS: 3 Critical thinking is essential for client care, particularly when the care is complex, involving numerous health issues. Following policies and procedures does not necessarily require critical thinking, and working with others or being flexible and working different schedules do not necessarily require critical thinking. PTS:1DIF:ModerateREF: p. 26-27; high-level question, answer not stated verbatim

A client newly diagnosed with diabetes is admitted to the hospital because her diabetes is out of control. Which of the following is an appropriate direct-care intervention for this client during her stay? 1) Consulting the diabetic nurse educator for help with a teaching plan 2) Making arrangements for the client to join a diabetic support group 3) Demonstrating blood glucose monitoring and insulin administration to the client 4) Consulting with the dietician about the client's dietary concerns

ANS: 3 Demonstrating blood glucose monitoring and insulin administration is an appropriate direct-care intervention for this client. Direct-care interventions are performed through intervention with the client and include interventions such as physical care, emotional support, and client teaching. Indirect-care activities include consulting the diabetic nurse educator, making arrangements for the client to join a diabetic support group, and consulting with the dietitian about the client's dietary concerns. Indirect-care activities are performed away from but on behalf of the client. PTS:1DIF:ModerateREF:p. 103 KEY: Nursing process: Interventions | Client need: Physiological integrity | Cognitive level: Application

The nurse is planning care for a patient. She is using a standardized care plan for Impaired Walking related to left-side weakness. Which of the following activities will the nurse perform when individualizing the plan for the patient? 1) Validate conflicting data with the patient. 2) Transcribe medical orders. 3) State the frequency for ambulation. 4) Perform a comprehensive assessment.

ANS: 3 Individualizing the care plan means identifying specific problems, outcomes, and interventions and the frequency of those interventions to meet the patient's needs. Validating data ensures your assessment is accurate. Transcribing orders is a part of developing and implementing the care plan but not of individualizing the plan. Performing an assessment is the beginning step to developing a care plan. Assessment helps you to know the ways in which a standardized plan needs to be individualized. PTS:1DIF:ModerateREF: p. 90 KEY: Nursing process: Planning | Client need: SECE | Cognitive level: Application

Nondirective interviewing is a useful technique because it 1) Allows the nurse to have control of the interview 2) Is an efficient way to interview a patient 3) Facilitates open communication 4) Helps focus patients who are anxious

ANS: 3 Nondirective interviewing helps build rapport and facilitates open communication. Because it puts the patient in control, it can be very time-consuming (inefficient) and produce information that is not relevant. Directive interviewing should be used to focus anxious patients. PTS:1DIF:EasyREF:p. 45 KEY: Nursing process: Assessment | Client need: PSI | Cognitive level: Recall

Which of the following most accurately describes nursing diagnoses? A nursing diagnosis 1) Supports the nurse's diagnostic reasoning 2) Supports the client's medical diagnosis 3) Identifies a client's response to a health problem 4) Identifies a client's health problem

ANS: 3 Nursing diagnoses are statements that nurses use to describe a client's physical, mental, emotional, spiritual, and social response to disease, injury, or other stressor. Diagnostic reasoning is used to identify the appropriate nursing diagnosis; it is not meant to "support" the diagnosis. A health problem is a condition that requires intervention to promote wellness or prevent illness; it is sometimes, but not always, a nursing diagnosis. Nursing diagnoses are not medical diagnoses. PTS:1DIF:ModerateREF: p. 57 KEY: Nursing process: Diagnosis | Client need: SECE | Cognitive level: Analysis

Which of the following is an example of an open-ended question? 1)Have you had surgery before? 2)When was your last menstrual period? 3)What happens when you have a headache? 4)Do you have a family history of heart disease?

ANS: 3 Open-ended questions such as "What happens when you have a headache?" are broad so as to encourage the patient to elaborate. The questions about surgery, menstrual period, and family history can all be answered with a "yes," "no," or short, specific answer (a date). PTS:1DIF:ModerateREF:pp. 45 KEY: Nursing process: Assessment | Client need: SECE | Cognitive level: Application

Which of the following is the best example of an outcome statement? The patient will 1) Use the incentive spirometer when awake 2) Walk two times during day and evening shifts 3) Maintain oxygen saturation above 92% while performing ADLs each morning 4) Tolerate 10 sets of range-of-motion exercises with physical therapy

ANS: 3 Outcome statements should have specific performance criteria and a target time; "maintain oxygen saturation" is the only one that meets those criteria. The incentive spirometer goal should say how many times the incentive spirometer should be used each hour as well as the volume. The walking goal should state how far the patient should walk. In the range-of-motion goal, tolerate is a vague word and is difficult to measure, and the outcome needs to specify how often. PTS:1DIF:ModerateREF: p. 91-92 KEY: Nursing process: Planning | Client need: SECE | Cognitive level: Analysis

Which of the following is an example of practical knowledge? (Assume all are true.) 1)The tricuspid valve is between the right atrium and ventricle of the heart. 2)The pancreas does not produce enough insulin in type 1 diabetes. 3)When assessing the abdomen, you should auscultate before palpating. 4)Research shows pain medication given intravenously acts faster than by other routes.

ANS: 3 Practical knowledge is knowing what to do and how to do it, such as how to do an assessment. The others are examples of theoretical knowledge, anatomy (tricuspid valve), fact (type 1 diabetes), and research (IV pain medication). PTS:1DIF:ModerateREF:p. 30; high-level question, answer not stated verbatim

Which of the following is an example of self-knowledge? The nurse thinks, "I know that I 1)Should take the client's apical pulse for 1 minute before giving digoxin" 2)Should follow the client's wishes even though it is not what I would want" 3)Have religious beliefs that may make it difficult to take care of some clients" 4)Need to honor the client's request not to discuss his health concern with the family"

ANS: 3 Self-knowledge is being aware of your religious and cultural beliefs and values. Taking the pulse is an example of practical knowledge. Following client wishes and honoring client requests are examples of ethical knowledge. PTS:1DIFgrinifficultREF: p. 30; high-level question, answer not stated verbatim | V1, p. 32; high-level question, answer not stated verbatim

Who is the primary decision maker when caring for healthy adult clients? 1) Physician 2) Family 3) Client 4) Nurse

ANS: 3 The client is the primary decision maker in the care of healthy clients. The nurse functions as a teacher and health counselor. The physician plays a role in health promotion and screening. The family may give input, but the client is the decision maker. PTS:1DIF:EasyREF: p. 110 KEY: Nursing process: Planning | Client need: Health promotion | Cognitive level: Comprehension

Which nursing diagnosis is written in the correct format when using the NANDA-I taxonomy? 1) Bowel Obstruction related to recent abdominal surgery A.M.B. nausea, vomiting, and abdominal pain 2) Inability to Ingest Food related to imbalanced nutrition: less than body requirements A.M.B. inadequate food intake, weight less than 20% under ideal body weight 3) Impaired Skin Integrity related to physical immobility A.M.B. skin tear over sacral area 4) Caregiver Role Strain related to alienation from family and friends A.M.B 24-hour care responsibilities

ANS: 3 The components of NANDA-I nursing diagnosis might include the following four parts: diagnostic label, defining characteristics, related factors, and risk factors. "Impaired Skin Integrity . . ." has the problem statement, etiology, and symptoms. For "Bowel Obstruction . . ." the problem is a medical diagnosis. The cause-and-effect order of "Inability to Ingest Food . . ." is incorrect; it starts with the etiology. The etiology and symptoms (A.M.B.) of "Caregiver Role Strain . . ." are reversed (alienation from family and friends are the symptoms that support the diagnosis). PTS: 1 DIF: Difficult REF: pp. 70-71 KEY: Nursing process: Diagnosis | Client need: SECE | Cognitive level: Application

Which criterion might be used in structure evaluation? 1) Staff refrains from sharing computer password. 2) Healthcare provider washes hands with each client contact. 3) A defibrillator is accessible on each client care area. 4) Nurse verifies client identification before initiating care.

ANS: 3 The criterion that states "a defibrillator is present on each client care area" is associated with structure evaluation. "Refrains from sharing computer password," "washes hands before each client contact," and "verifies client identification before initiating care" are criteria associated with process evaluation. PTS:1DIF:ModerateREF: p. 127 KEY: Nursing process: Evaluation | Client need: SECE | Cognitive level: Analysis

The nurse is beginning discharge planning for an older adult with left-side weakness. All of the following are important, but which action is most important in ensuring that the discharge plan is successful? 1) Start planning at admission. 2) Involve the family members. 3) Get patient input when making the plan. 4) Involve the multidisciplinary team.

ANS: 3 The discharge plan may be developed in a timely manner and involve the family and a multidisciplinary team, but if the patient does not agree with the plan, it will not be successful. PTS: 1 DIF: Moderate REF: pp. 83| p. 87 KEY: Nursing process: Planning | Client need: SECE | Cognitive level: Analysis

Which of the following explains why it is important to have the correct etiology for a nursing diagnosis? The etiology 1) Is the cause of the problem 2) Cannot always be observed 3) Directs nursing care 4) Is an inference

ANS: 3 The etiology directs nursing interventions. If the incorrect etiology is given, the nursing care would not be appropriate for the client. The other statements are true but not a reason for the importance of the etiology being correct. PTS:1DIFgrinifficultREF:pp. 63 KEY: Nursing process: Diagnosis | Client need: SECE | Cognitive level: Analysis

Which nursing diagnosis is written in the correct format? 1) Imbalanced Nutrition: Less than Body Requirements related to body weight less than 20% under ideal weight 2) Ineffective Airway Clearance related to increased respiratory rate and irregular rhythm 3) Impaired Swallowing related to absent gag reflex 4) Excess Fluid Volume related to 3 lb weight gain in 24 hours

ANS: 3 The etiology should describe what is causing or contributing to the problem. The etiologies for Ineffective Airway Clearance, Impaired Airway Swallowing, and Excess Fluid Volume describe signs or symptoms rather than causal factors. PTS: 1 DIF: Difficult REF: V1, p. 64 | pp. 70-73 KEY: Nursing process: Diagnosis | Client need: SECE | Cognitive level: Analysis

The nurse has just finished documenting that he removed a patient's nasogastric tube. Which is the next logical step in the nursing process? 1) Assessment 2) Planning 3) Evaluation 4) Diagnosis

ANS: 3 The implementation phase ends when you document nursing actions on the client's chart. Implementation evolves into the evaluation step when you document the client's response to your interventions. As a general rule, the steps in order are as follows: assessment diagnosis, planning outcomes, planning interventions, implementation, and evaluation. PTS:1DIF:EasyREF:p. 125 KEY: Nursing process: Implementation | Client need: SECE | Cognitive level: Comprehension

Which situation is the most conducive to conducting a successful interview of an elderly woman whose husband and two children are in the hospital room visiting and watching television? The woman is alert and oriented. 1) Provide enough chairs so the family and you are able to sit facing the client. 2) Introduce yourself and ask, "Dear, what name do you prefer to go by?" before asking any questions. 3) After the family leaves, ask the client if she is comfortable and willing to answer a few questions. 4) Ask the client if you can talk with her while her family is watching the television.

ANS: 3 The interview should be done when the client is comfortable and there are no distractions. Endearing terms are inappropriate unless the client prefers them. Family members may offer information that may or may not be pertinent and may distract from the interview. The presence of family members may also inhibit full disclosure of information by the client. PTS:1DIFgrinifficultREF:p. 46-47 KEY: Nursing process: Assessment | Client need: PSI | Cognitive level: Application

The nurse is performing preoperative teaching for a client who is scheduled for surgery in the morning. The client does not currently have any respiratory problems. The nurse's teaching plan includes coughing and deep breathing exercises. Which type of nursing intervention is the nurse performing? 1) Health promotion 2) Treatment 3) Prevention 4) Assessment

ANS: 3 The nurse is teaching the client coughing and deep breathing exercises, which help prevent postoperative pneumonia. Therefore, the nurse is utilizing a prevention intervention. Prevention interventions are used to help prevent complications, such as postoperative pneumonia. Health-promotion interventions promote a client's efforts to achieve a higher level of wellness. Treatment interventions treat disorders, relieve symptoms, and carry out medical orders. Assessment interventions detect changes in the client's condition and detect potential problems. PTS:1DIF:ModerateREF: p. 106 KEY: Nursing process: Interventions | Client need: Physiological integrity | Cognitive level: Application

A physician prescribes oral aripiprazole 10 mg daily for a client with schizophrenia. This medication is unfamiliar to the nurse, and she cannot find it in the hospital formulary or other references. How should she proceed? 1) Administer the medication as prescribed. 2) Hold the medication and notify the prescriber. 3) Consult with a pharmacist before administering it. 4) Ask the patient's nurse for information about the medication.

ANS: 3 The nurse must recognize when she does not have the knowledge or skill needed to implement an order. Because the nurse is unfamiliar with the medication, that does not mean she should hold it and delay patient treatment. It is wisest to first consult with the pharmacist for information before administering the medication to ensure safe practice. Administering the medication as prescribed, without knowing its expected actions and side effects, at the least prevents adequate reassessment; at the most, it is dangerous. Holding the medication and notifying the prescriber prevents the client from receiving timely treatment—many drugs are less effective if a consistent schedule is not maintained. Asking another nurse to administer the medication is also unsafe because it cannot be assumed that the other nurse has the correct knowledge. In addition, the nurse caring for the client must assess for adverse reactions to the medication. PTS:1DIF:ModerateREF:p. 118 KEY: Nursing process: Implementation | Client need: SECE | Cognitive level: Analysis

How does nursing diagnosis differ from a medical diagnosis? A nursing diagnosis is 1)Terminology for the client's disease or injury 2)A part of the client's medical diagnosis 3)The client's presenting signs and symptoms 4)A client's response to a health problem

ANS: 4 A nursing diagnosis is the client's response to actual or potential health problems. PTS:1DIF:ModerateREF: p. 31 KEY: Nursing process: Diagnosis | Client need: SECE | Cognitive level: Recall

A nursing instructor is guiding nursing students on best practices for interviewing patients. Which of the following comments by a student would indicate the need for further instruction? 1) "My patient is a young adult, so I plan to talk to her without her parents in the room." 2) "Because my patient is old enough to be my grandfather, I will call him 'Mr.'" 3) "When reading my patient's health record, I thought of a few questions to ask." 4) "When I give my patient his pain medication, I will have time to ask questions."

ANS: 4 A patient should be comfortable when interviewing. The pain medication should have time to work before considering interviewing the patient, so asking questions when giving the medication is not a good idea. It is appropriate to interview patients without family/friends around. In nearly every culture, calling a patient Mr. or Mrs. shows respect and is therefore correct. Reading the patient's health record is appropriate preparation for an interview. PTS:1DIF:ModerateREF:p. 46 KEY: Nursing process: Evaluation | Client need: SECE | Cognitive level: Application

Of the following recommended interviewing techniques, which one is the most basic? (That is, without that intervention, the others will all be less effective.) 1)Beginning with neutral topics 2)Individualizing your approach 3)Minimizing note taking 4)Using active listening

ANS: 4 All are important techniques, but active listening focuses the attention on the patient and lets her know you are trying to understand her needs. The interviewer is more likely to get the patient to open up. Patients will forgive you for most errors in technique, but if they think you are not listening, that can negatively affect your relationship. PTS:1DIFgrinifficultREF:47

Which of the following describes the most important use of nursing diagnosis? (All statements are true.) 1) Differentiates the nurse's role from that of the physician 2) Identifies a body of knowledge unique to nursing 3) Helps nursing develop a more professional image 4) Describes the client's needs for nursing care

ANS: 4 Benefits to nurses and nursing are that nursing diagnoses differentiate the nurse's role, they identify a unique body of nursing knowledge, and some think they help nursing to develop a more professional image. However, the primary goal of nursing is to serve the good of the patient. Therefore, the most important use of a diagnosis is to specifically identify the client's needs for quality nursing care. PTS: 1 DIF: Moderate REF: p. 56 KEY: Nursing process: Diagnosis | Client need: Safe-care environment | Cognitive level: Analysis

Which of the following describes the difference between a collaborative problem and a medical diagnosis? 1) A collaborative problem is treated by the nurse; a physician is responsible for the treatment of a medical problem. 2) A collaborative problem is a nursing diagnosis that requires specific orders from a physician; a medical diagnosis directs all nursing care. 3) A collaborative problem has the potential to become an actual nursing diagnosis; a medical diagnosis rarely changes. 4) A collaborative problem requires intervention by the nurse and physician or other professional; a medical diagnosis requires intervention by a physician.

ANS: 4 Collaborative problems are physiological complications a client may be at risk for due to her medical diagnosis, medical treatment, or diagnostic studies. A collaborative problem requires monitoring by the nurse and intervention by a physician. A medical diagnosis requires interventions (medications, treatments) by the physician. Medical diagnoses do not direct all nursing care. Collaborative problems have the potential to become medical, not nursing, diagnoses. PTS:1DIF:ModerateREF: pp. 58-59 KEY: Nursing process: Diagnosis | Client need: SECE | Cognitive level: Analysis

Which client has the greatest need for comprehensive discharge planning? 1) A woman who has just given birth to her second child and lives with her husband and 18-month-old daughter 2) A man who has been readmitted for exacerbation of his chronic obstructive pulmonary disease 3) A 12-year-old boy who had outpatient surgery on his knee and lives with his mother 4) A woman who was just diagnosed with renal failure and has started peritoneal dialysis

ANS: 4 Comprehensive discharge planning should be done for patients who have a newly diagnosed chronic disease or have complex needs. The other patients may require discharge planning but not as comprehensive as someone with a new diagnosis with complex treatment. PTS:1DIFgrinifficultREF: p. 83 KEY: Nursing process: Planning | Client need: SECE | Cognitive level: Analysis

The nurse is caring for a client who was newly diagnosed with type 2 diabetes mellitus. Which intervention by the nurse best promotes client cooperation with the treatment plan? 1) Teaching the client that he must lose weight to control his blood sugar 2) Informing the client he must exercise at least three times per week 3) Explaining to the client that he must come to the diabetic clinic weekly 4) Determining the client's main concerns about his diabetes

ANS: 4 Determining the client's main concerns promotes cooperation with the treatment regimen. For example, if the client is concerned about paying for diabetic monitoring equipment, he may disregard any teaching about the procedure. Although it is often important for a diabetic client to exercise and lose weight to control blood sugar levels, the client must want to do both. He will not exercise or lose weight simply because he is told to do so. The nurse must assess the client's support systems and resources, not just tell him he must come to the diabetic clinic weekly. Some clients do not have access to transportation and, therefore, could not come to the clinic without social service intervention. Remember that knowledge does not necessarily change behavior. PTS:1DIF:ModerateREF: p. 122 KEY: Nursing process: Planning interventions | Client need: PHSI | Cognitive level: Analysis

Why is it important to obtain information about nutritional and herbal supplements as well as about complementary and alternative therapies? 1) To determine what type of therapies are acceptable to the client 2) To identify whether the client has a nutrition deficiency 3) To help you to understand cultural and spiritual beliefs 4) To identify potential interaction with prescribed medication and therapies

ANS: 4 Herbs and nutritional supplements can interact with prescription medications, and complementary and alternative treatments can interfere with conventional therapies. Physical assessment and laboratory tests are needed to assess a nutritional deficiency. To identify cultural and spiritual beliefs and well as what therapies are acceptable to the client, you need more than just information about nutritional and herbal supplements. PTS:1DIFgrinifficultREF: p. 45

Which intervention depends almost entirely on the client's adhering to the therapy? 1) Inserting an IV catheter 2) Turning a client every 2 hours 3) Shortening a surgical drain 4) Following a low-fat, low-calorie diet

ANS: 4 Instituting and adhering to a low-fat, low-calorie diet is an intervention that depends almost entirely on the client's adhering to the therapy. Client cooperation is necessary for performing the other interventions, but the interventions do not depend on the client to the same extent. PTS:1DIF:EasyREF: p. 122 KEY: Nursing process: Planning interventions | Client need: SECE | Cognitive level: Analysis

The nurse administering pain medication every 4 hours is an example of which aspect of patient care? 1)Assessment data 2)Nursing diagnosis 3)Patient outcome 4)Nursing intervention

ANS: 4 Interventions are activities that will help the patient achieve a goal, such as administering pain-relieving medication. An example of assessment data might be, "Patient reports pain is a 5 on a 1 to 10 scale." The nursing diagnosis would be Pain. The nurse might define the patient outcome in this scenario as, "The patient will state the level of pain is less than 4." PTS:1DIF:ModerateREF:p. 31; high-level question, answer not stated verbatim KEY: Nursing process: Interventions | Client need: SECE | Cognitive level: Application

Which of the following outcome statements contains the best example of performance criteria? The patient will 1) Turn herself in bed frequently while awake 2) Understand how to use crutches by day 2 3) State that pain is decreased after being medicated 4) Eat 75% of each meal without complaint of nausea

ANS: 4 Performance criteria should be specific and measurable. "75% of each meal" is specific and measurable. "Frequently" is vague. You cannot observe whether someone "understands." "Decreased" is vague; a numerical pain rating would be better. PTS:1DIF:ModerateREF: p. 92 KEY: Nursing process: Planning | Client need: SECE | Cognitive level: Application

Which nursing intervention is best individualized to meet the needs of a specific client? 1) Suction the client every 2 hours per unit policy. 2) Use incentive spirometry every hour while awake per postoperative protocols. 3) Institute swallowing precautions. 4) Move client out of bed to the chair daily; client prefers to be out of bed for dinner.

ANS: 4 Positioning the client in the chair for meals considers the client's desire to be out of bed for dinner, so it is obviously individualized. An intervention performed according to unit policy or protocols is not necessarily individualized. "Institute swallowing precautions" does not provide instructions for the specific actions needed to do that for "this particular" client. PTS:1DIF:ModerateREF:p. 118; high-level question, answer not given verbatim KEY: Nursing process: Planning interventions | Client need: SECE | Cognitive level: Application

Which of the following is a benefit of standardized care plans, as defined in your text? Standardized care plans 1) Apply to every patient on a particular unit 2) Include both medical and nursing orders 3) Specify patient outcomes for each day 4) Help ensure that important interventions are not overlooked

ANS: 4 Standardized care plans help promote consistency of care and ensure that important interventions are not forgotten. They are not likely to apply to every patient on a unit because they are usually single-problem plans or are used with a particular medical diagnosis. Unlike protocols, they do not include medical orders. Unlike critical pathways, they do not specify predicted patient outcomes for each day. PTS:1DIF:ModerateREF: p. 86 KEY: Nursing process: Planning | Client need: SECE | Cognitive level: Recall

What is the role of the Joint Commission in regard to patient assessment? The Joint Commission 1)States what assessments are collected by individuals with different credentials 2)Regulates the time frames for when assessments should be completed 3)Identifies how data are to be collected and documented 4)Sets standards for what and when to assess the patients

ANS: 4 The Joint Commission sets detailed standards regarding what and when to assess but does not address credentials. Nurse practice acts specify what data are collected and by whom. Agency policy may set time frames for when assessments should be done and how they should be documented. Nursing knowledge identifies "how" data are to be collected. PTS:1DIF:ModerateREF:p. 39

When making a diagnosis using NANDA-I, which of the following provides support for the diagnostic label you choose? 1) Etiology 2) Related factors 3) Diagnostic label 4) Defining characteristics

ANS: 4 The defining characteristics are the signs and symptoms that must be present to support any given nursing diagnosis. The etiology and related factors are the causes or contributing factors to the problem. The diagnostic label is the name NANDA-I has given the problem; it is chosen based on the presence of defining characteristics. PTS:1DIF:EasyREF:p. 68 KEY: Nursing process: Diagnosis | Client need: SECE | Cognitive level: Recall

When should the nurse collect evaluation data for this expected outcome: Patient will maintain urine output of at least 30 mL/hour? 1) At the end of the shift 2) Every 24 hours 3) Every 4 hours 4) Every hour

ANS: 4 The nurse should collect evaluation data as defined in the expected outcome. For instance, in this case, the nurse would check the patient's urine output every hour because the goal statement specifies an hourly rate (30 mL/hour). The unit of measurement in the goal guides how often the nurse would reassess the patient. PTS:1DIF:EasyREF: pp. 127-128 KEY: Nursing process: Evaluation | Client need: PHSI | Cognitive level: Application

When should the nurse make systematic observations about a patient? 1)When the patient has specific complaints 2)With the first assessment of the shift 3)Each time the nurse gives medications to the patient 4)Each time the nurse interacts with the patient

ANS: 4 The nurse should make observations about the patient each time she enters the room or interacts with the patient to gain ongoing data about the patient. PTS:1DIF:EasyREF:p. 41-42 KEY: Nursing process: Assessment | Client need: SECE | Cognitive level: Application

Which of the following is an example of data that should be validated? 1)The client's weight measures 185 lb at the clinic. 2)The client's liver function test results are elevated. 3)The client's blood pressure is 160/94 mm Hg; he states that that is typical for him. 4)The client states she eats a low-sodium diet and reports eating processed food.

ANS: 4 Validation should be done when the client's statements are inconsistent (processed foods are generally high in sodium). Validation is not necessary for laboratory data when you suspect an error has been made in the results. Personal information that patients might be embarrassed about, such as weight, is best validated with a scale. PTS:1DIF:ModerateREF: p. 47 KEY: Nursing process: Assessment | Client need: PHSI | Cognitive level: Application

A nursing instructor asked his nursing students to discuss their experiences with charting assessment data. Which comment by the student indicates the need for further teaching? 1) "I find it difficult to avoid using phrases like, 'The patient tolerated the procedure well.'" 2) "It's confusing to have to remember which abbreviations this hospital allows." 3) "I need to work on charting assessments and interventions right after they are done." 4) "My patient was really quiet and didn't say much, so I charted that he acted depressed."

ANS: 4 When charting data, chart only what was observed, not what it meant. Inferences should not be made about a patient's behavior during data collection ("he acted depressed"); so that response reflects the student's lack of knowledge and need for teaching. Chart specific data, not vague phrases; the student is acknowledging the importance of this. There are no universally accepted phrases, just agency-approved abbreviations; the student is acknowledging the need to use agency-approved abbreviations. The student is correct that charting should be completed as soon after data collection as possible. PTS:1DIF:ModerateREF:p. 50 KEY: Nursing process: Evaluation | Client need: SECE | Cognitive level: Application

A nurse makes a nursing diagnosis of Acute Pain related to the postoperative abdominal incision. She writes a nursing order to reposition the client in a comfortable position using pillows to splint or support the painful areas. What type of nursing intervention did the nurse write? 1) Collaborative 2) Interdependent 3) Dependent 4) Independent

ANS: 4 Writing an order to reposition the client in a comfortable position is an example of an independent nursing intervention, one that does not require a physician's order. The nurse is licensed to prescribe, perform, or delegate the intervention based on her knowledge and skills. A collaborative or interdependent intervention is one that is carried out in collaboration with other health team members, such as providing the client with a sodium-restricted diet. A dependent intervention is prescribed by a physician or advanced practice nurse; for example, administer oxygen at 2 L/min via nasal cannula. PTS:1DIF:ModerateREF: p. 103 KEY: Nursing process: Interventions | Client need: Physiological integrity | Cognitive level: Application

The client's weight is appropriate for his height. His laboratory values and other assessments reflect normal nutritional status. However, he has told the nurse, "I probably eat a little too much red meat. And what is this I hear about needing omega 3 oils in my diet? I don't like to take supplements, and I think I could really improve my nutrition." Which of the following nursing diagnoses should the nurse use? 1) Balanced Nutrition 2) Possible Imbalanced Nutrition: Less Than Body Requirements 3) Risk for Imbalanced Nutrition: Less Than Body Requirements 4) Readiness for Enhanced Nutrition

ANS: 4 You will use a wellness diagnosis when a person's present level of wellness is effective and when the person wants to move to a higher level of wellness—in this case, a higher level of nutrition. The format for a wellness diagnosis is "Readiness for Enhanced . . ." Use a possible diagnosis when you have enough data to suspect a problem but need more data to support a diagnosis. Use a risk diagnosis when there are risk factors for a problem. PTS: 1 DIF: Moderate REF: p. 72 KEY: Nursing process: Diagnosis | Client need: SECE | Cognitive level: Application

Match the assessment model with the intended use for that model. 1) Categorizes nursing diagnoses, client outcomes, and nursing interventions 2) Assesses the client's ability to achieve balance (homeostasis) 3) Identifies deficits in activities of daily living that require nursing assistance 4) Formulates a model for nursing assessment and diagnosis but is not a theory 5) Categorizes nursing diagnoses ____ 12. Gordon's Functional Health Patterns ____ 13. NANDA Nursing Diagnosis Taxonomy II ____14.Taxonomy of Nursing Practice ____ 15. Roy's Adaptation model ____ 16. Orem's Self-Care model

ANS: 4 PTS: 1 DIF: Moderate REF: p. 49 KEY: Nursing process: Assessment | Client need: SECE | Cognitive level: Recall 13. ANS: 5 PTS: 1 DIF: Moderate REF: p. 49 KEY: Nursing process: Assessment | Client need: SECE | Cognitive level: Recall 14. ANS: 1 PTS: 1 DIF: Moderate REF: p. 49 KEY: Nursing process: Assessment | Client need: SECE | Cognitive level: Recall 15. ANS: 2 PTS: 1 DIF: Moderate REF: p. 49 KEY: Nursing process: Assessment | Client need: SECE | Cognitive level: Recall 16. ANS: 3 PTS: 1 DIF: Moderate REF: p. 49 KEY: Nursing process: Assessment | Client need: SECE | Cognitive level: Recall

Using Maslow's hierarchy of needs, rank the following nursing diagnoses in order of importance, beginning with the highest-priority diagnosis. 1) Anxiety 2) Risk for infection 3) Disturbed body image 4) Sleep deprivation

ANS: 4, 2, 1, 3 In Maslow's hierarchy, physiologic needs and safety are the highest priority. Sleep is a basic physiologic need. Infection can threaten physical health. In this question, infection is not present; therefore, there is just a risk for it. Sleep Deprivation is an immediate problem that affects general physical, mental, and emotional health. Neither Anxiety nor Disturbed Body image is a physiologic or safety need. Anxiety is a more immediate need than Disturbed Body Image, so it probably deserves a higher ranking. Remind students that the ranking would depend on the severity of each problem, which is not known by the labels alone. PTS:1DIFgrinifficultREF:pp. 64 KEY: Nursing process: Diagnosis | Client need: SECE | Cognitive level: Analysis

The nurse should avoid asking the client which of the following leading questions during a client interview? A. "What medication do you take at home?" B. "You are really excited about the plastic surgery, aren't you?" C. "Were you aware I've has this same type of surgery?" D. "What would you like to talk about?"

B. "You are really excited about the plastic surgery, aren't you?" Rationale: A leading question directs the client's answer. The phrasing of the question indicates an expected answer. The client may be influenced by the nurse's expectations and may give inaccurate responses. This process can result in an error in diagnostic reasoning.

The nurse would do which of the following activities during the diagnosing phase of the nursing process? Select all that apply. A. Collect and organize client information B. Analyze data C. Identify problems, risk, and client strengths D. Develop nursing diagnoses E. Develop client goals

B. Analyze data C. Identify problems, risk, and client strengths D. Develop nursing diagnoses Rationale: The diagnosing phase of the nursing process involves data analysis, which leads to identification of problems, risks, and strengths and the development of nursing diagnoses. Collecting and organizing client data is done in the assessment phase of the nursing process. Goal setting occurs during the planning phase.

Which of the following descriptors is most appropriate to use when stating the "problem" part of a nursing diagnosis? A. Grimacing B. Anxiety C. Oxygenation saturation 93% D. Output 500 mL in 8 hours

B. Anxiety Rationale: The problem part of a nursing diagnosis should state the client's response to a life process, event, or stressor. These are categorized as nursing diagnoses. The incorrect options are cues the nurse would use to formulate the nursing diagnostic statement.

The nurse questions if the dosage of a medication is unsafe for the client because of the client's weight and age. The nurse should take which of the following actions? A. Administer the medication as ordered by the prescriber B. Call the prescriber to discuss the order and the nurse's concern C. Administer the medication, but chart the nurse's concern about the dosage D. Give the client half the dosage and document accordingly

B. Call the prescriber to discuss the order and the nurse's concern Rationale: Client safety is of the utmost importance when implementing any nursing intervention. If the nurse feels that an order is unsafe or inappropriate for a client, the nurse must act as a client advocate and collaborate with the appropriate healthcare team member to determine the rationale for the order and/or modify the order as necessary. A nurse accepts accountability for his or her actions. Options 1, 3, and 4 are inappropriate and unsafe.

The functional health pattern assessment data states: "Eats three meals a day and is of normal weight for height." The nurse should draw which of the following conclusions about this data? Select all that apply. A. Client has an actual health problem B. Client has a wellness diagnosis C. Collaborative health problem needs to be written D. Possible nursing diagnosis exists E. Specific questions about the diet should be asked next

B. Client has a wellness diagnosis E. Specific questions about the diet should be asked next Rationale: The description indicates a healthy pattern of nutrition for the client. A wellness diagnosis might be stated as: "Potential for enhanced nutrition." An actual health problem is a client problem that is currently present. The nurse should also do a diet assessment to determine the quality of the food eaten during meals. These actions by the nurse are within the scope of independent nursing practice and are not collaborative in nature.

The client reports nausea and constipation. Which of the following would be the priority nursing action? A. Collect a stool sample B. Complete an abnormal assessment C. Administer an anti-nausea medication D. Notify the physician

B. Complete an Abdominal assessment Rationale: Assessment involves the systematic collection of data about an individual upon which all subsequent phases of the nursing process are built. In response to a client's complaint, a nurse assesses a specific body system to obtain data that will help the nurse make a nursing diagnosis and plan the client's care. The other options reflect interventions, which are not timely unless there is first a complete assessment.

The client is being discharged to a long-term care (LTC) facility. The nurse is preparing a progress note to communicate to the LTC staff the client's outcome goals that were met and those that were not. To do this effectively, the nurse should: A. Formulate post-discharge nursing diagnoses B. Draw conclusion about resolution of current client problems C. Assess the client for baseline data to be used at the LTC facility D. Plan the care that is needed in the LTC facility

B. Draw conclusion about resolution of current client problems Rationale: Terminal evaluation is done to determine the client's condition at the time of discharge. This evaluation is best reflected in option 2 because it focuses on which goals were achieved and which were not. Ongoing evaluation is done while or immediately after implementing a nursing intervention. Intermittent evaluation is performed at specified intervals, such as twice a week. Items related to care post-discharge (options 2, 3, and 4) should be done on admission to the LTC facility.

The nurse would place which correctly written nursing diagnostic statement into the client's care plan? A. Cancer relater to cigarette smoking B. Impaired gas exchange related to aspiration of foreign matter as evidenced by oxygen saturation of 91% C. Imbalance nutrition: more than body requirement related to overweight status D. Impaired physical mobility related to generalized weakness and pain

B. Impaired gas exchange related to aspiration of foreign matter as evidence by oxygen saturation of 91% Rationale: A nursing diagnosis consists of two parts joined by related to. The first part (the human response) names/labels the problem. The second part (related factors) includes the factors that either contribute to or are probable etiologies of the human response. Some formats include a third part to the statement for actual (not risk) diagnoses; this third part consists of the client's signs or symptoms and is joined to the statement with the label as evidenced by. This type of statement is the most complete. Option 1 is not a nursing diagnosis but is a medical diagnosis. Options 3 and 4 are vague.

The nurse who documents on the client's care plan the outcome goal "Anxiety will be relieved within 20 to 40 minutes following administration of lorazepam (Ativan)" is engaged in which step of the nursing process? A. Assessment B. Planning C. Implementation D. Evaluation

B. Planning Rationale: The planning step of the nursing process involves formulating client goals and designing the nursing interventions required to prevent, reduce, or eliminate the client's health problems. Outcome goals are documented on the client's care plan. Assessment data (option 1) is used to help identify a client's human response, and once a plan is established, the interventions are implemented (option 3) and evaluated (option 4).

The nurse would do which of the following during the implementation phase of the nursing process when working with a hospitalized adult? A. Formulate a nursing diagnosis of impaired gas exchange B. Record in the medical record the distance a client ambulate in the hall C. Write individualized nursing orders in the care plan D. Compare client responses to the desired outcomes for pain relief

B. Record in the medical record the distance a client ambulate in the hall Rationale: The implementation phase of the nursing process involves carrying out or delegating the nursing interventions and recording nursing activities and client responses in the medical records. Option 1 represents diagnosing. Option 3 represents planning. Option 4 represents evaluation.

The nurse makes the following entry on the client's care plan: "Goal not met. Client refuses to ambulate, stating, 'I am too afraid I will fall.' " The nurse should take which of the following actions? A. Notify the physician B. Reassign the client to another nurse C. Reexamine the nursing orders D. Write a new nursing diagnosis

B. Reexamine the nursing orders Rationale: The plan needs to be reassessed whenever goals are not met. Nursing interventions should be examined to ensure the best interventions were selected to assist the client achieve the goal. The goal may be appropriate, but the client may need more time to achieve the desired outcome. The manner in which the nursing interventions were implemented may have interfered with achieving the outcome.

The nursing diagnosis is Risk for impaired skin integrity related to immobility and pressure secondary to pain and presence of a cast. Which of the following desired outcomes should the nurse include in the care plan? A. Client will be able to turn self by day 3 B. Skin will remain intact and without redness during hospital stay C. Client will state pain relieved within 30 minutes after medication D. Pressure will be prevented by repositioning client every 2 hours

B. Skin will remain intact and without redness during hospital stay Rationale: The human response/label is what needs to change (Risk for impaired skin integrity). The label suggests the outcomes. In this case, "skin will remain intact" is the desired outcome for a client at risk for impaired skin integrity. Option 1 addresses immobility. Option 3 addresses pain. Option 4 is an intervention.

When the client resists taking a liquid medication that is essential to treatment, the nurse demonstrates critical thinking by doing which of the following first? A. Omitting this dose of medication and waiting until the client is more cooperative B. Suggesting the medication can be diluted in a beverage C. Asking the nurse manager about how to approach the situation D. Notifying the physician inability to give the client this medication

B. Suggesting the medication can be diluted in a beverage Rationale: Diluting the medication in a beverage may make the medication more palatable. Using critical thinking skills, the nurse should try to problem-solve in a situation such as this before asking for the assistance of the nurse manager. Suggesting an alternative method of taking the medication (provided that there are no contraindications to diluting the medication) should improve the likelihood of the client taking the medication.

Which desired outcome written by the nurse is correctly written and measurable? A. Client will have a normal bowel pattern by April 2 B. The client will lose 4 lbs. within next 2 weeks C. The nurse will provide skin care at least 3 times each day D. The client will breathe better after resting for 10 minutes

B. The client will lose 4 lbs. within next 2 weeks Rationale: An outcome statement must describe the observable client behavior that should occur in response to the nursing interventions. It consists of a subject, action verb, conditions under which the behavior is to be performed, and the level at which the client will perform the desired behavior. Each of the incorrect options lacks one of these required elements. Option 1 is not measurable. Option 3 is a nursing goal rather than a client goal. Option 4 does not include the level at which the behavior should be performed.

The nurse is measuring the client's urine output and straining the urine to assess for stones. Which of the following should the nurse record as objective data? A. The client reports abdominal pain B. The client's urine output was 450 mL C. The client states, "I didn't see any stones in my urine." D. The client states, "I feel like I have passed a stone."

B. The client's urine output was 450 mL. Rationale: Objective data is measurable data that can be seen, heard, or verified by the nurse. The objective data is the measurement of the urine output. A client's statements and reports of symptoms are documented as subjective data, such as the data found in options 1, 3, and 4.

Which professionally appropriate response should the nurse make when a more stringent policy for the use of restraints is introduced on a surgical unit? A. Use the previous, less restrictive policy conscientiously B. Express immediate disagreement with the new policy C. Ask for the rationale behind the new policy D. Obey the policy but continue to voice disapproval of it to co-workers

C. Ask for the rationale behind the new policy Rationale: Understanding the rationale behind a decision helps the nurse analyze the proposed change and understand its purpose. Options 1, 2, and 4 represent unprofessional behavior. Option 1 also places a client's safety at risk.

Which of the following outcome goals has the nurse designed correctly for the postoperative client's plan of care? Select all that apply. A. Client will state pain is less than or equal to 3 on zero to ten pain scale B. Client will have no pain C. Client will state pain is less than or equal to a 3 on a 0-10 pain scale within 24 hours D. Client will state pain is less than or equal to a 5 on a 0-10 pain scale by the time of discharge E. Client will be medicated every 4 hours by the nurse

C. Client will state pain is less than or equal to a 3 on a 0-10 pain scale within 24 hours D. Client will state pain is less than or equal to a 5 on a 0-10 pain scale by the time of discharge Rationale: An outcome goal should be SMART: specific, measurable, appropriate, realistic, and timely. Options 3 and 4 are SMART goals. Options 1 and 2 have no timeframe to achieve the goal and are therefore incomplete. Option 2 is also unrealistic; the nurse cannot expect a postoperative client to be pain free. Option 5 is not a client goal.

A client who complains of nausea and seems anxious is admitted to the nursing unit. The nurse should take which of the following actions regarding completion of the admission interview? A. Help the client to get settled and do the interview the next morning when the client is rested B. Do the interview immediately, directing the majority of the questions to the client's spouse C. Do the interview as soon as some uninterrupted time is available in order to address the client's concerns D. Ask the charge nurse to interview the client while the admitting nurse calls the doctor for anti-nausea and anti-anxiety medication

C. Do the interview as soon as some uninterrupted time is available in order to address the client's concerns Rationale: To collect data accurately, the client must participate. Attending to the client's immediate personal needs before expecting the client to focus on the interview will maximize the accuracy of the data collected. Data should be collected shortly after admission. The best source of data is the client. The management of the client's anxiety is the responsibility of the nurse conducting the interview and initiating the relationship.

A desired outcome for a client immobilized in a long leg cast reads; Client will state three signs of impaired circulation prior to discharge. When the nurse evaluates the client's progress, the client is able to state that numbness and tingling are signs of impaired circulation. What would be an appropriate evaluation statement for the nurse to write? A. Client understands the signs of impaired circulation B. Goal met: Client cited numbness and tingling as sign of impaired circulation C. Goal not met: Client able to name only two signs of impaired circulation D. Goal not met: Client unable to describe signs of impaired circulation

C. Goal not met: Client able to name only two signs of impaired circulation Rationale: The goal has not been met because the client states only two out of three signs of impaired circulation. By comparing the data with the expected outcomes, the nurse judges that while there has been progress toward the goal, it has not been completely met. The care plan may need to be revised or more effective teaching strategies may need to be implemented to achieve the goal.

The nurse informs the physical therapy department that the client is too weak to use a walker and needs to be transported by wheelchair. Which step of the nursing process is the nurse engaged in at this time? A. Assessment B. Planning C. Implementation D. Evaluation

C. Implementation Rationale: The nurse is responsible for coordinating the plan of care with other disciplines to ensure the client's safety. This action represents the implementation phase of the nursing process. Data gathering occurs during assessment. Goal setting occurs during planning. Determining attainment of client goals occurs as part of evaluation.

In developing a plan of care for a client with chronic hypertension, which nursing activity would be most important? A. Set incremental goals for blood pressure reduction B. Instruct the client to make dietary changes by reducing sodium intake C. Include the client and family when setting goals and formulating the plan of care D. Assess past compliance to medication regimens

C. Include the client and family when setting goals and formulating the plan of care Rationale: In developing a plan of care, nurses engage in a partnership with the client and family. Nurses do not plan care for clients; instead they plan care with clients and families. Assessment (option 4), goal setting (option 1), and interventions (option 2) will be most accurate and effective when carried out in partnership with the client and family. The other options represent other actions to take, but they will have less overall effectiveness if the client and family are not part of the plan.

The nurse overhears an unlicensed assistive person (UAP) who has just been accepted to nursing school say to a client, "You must be so pleased with your progress." The nurse later explains to the UAP that this is an example of what type of question? A. Close-ended question B. Open-ended question C. Leading question D. Neutral question

C. Leading question Rationale: A leading question is asked in a way that suggests the type of answer that is expected. This can result in inaccurate data collection. A closed-ended question generally requires only a "yes" or "no" or short factual answer. Open-ended questions encourage clients to elaborate on their thoughts and feelings. Neutral questions do not influence the client's answer.

The nurse feels a client is at risk for skin breakdown because he has only had clear liquids for the last 10 days (and essentially no protein intake). The nurse would formulate which diagnostic statement that would best reflect this problem? A. Risk for malnutrition related to clear liquid diet B. Impaired skin integrity related to no protein intake C. Risk for impaired skin integrity related to malnutrition D. Impaired nutrition related to current illness

C. Risk for impaired skin integrity related to malnutrition Rationale: This is a risk diagnosis, and the diagnostic statement has two parts: the human response (impaired skin integrity) and the related/risk factor (malnutrition). Options 1 and 2 do not have related factors that are under the control of the nurse (i.e., type of diet ordered). The diagnosis in option 4 does not specify the type of impairment (greater than or less than body requirements) and is therefore incomplete. It also does not provide direction for development of goals and interventions.

The nurse needs to validate which of the following statements pertaining to an assigned client? A. The client has a hard, raised, red lesion on his right hand. B. A weight of 185 lbs. is recorded in the chart C. The client reported an infected toe D. The client's blood pressure is 124/70. It was 118/68 yesterday.

C. The client reported an infected tow Rationale: Validation is the process of confirming that data are actual and factual. Data that can be measured can be accepted as factual, as in options 1, 3 and 4. The nurse should assess the client's toe to validate the statement.

Which nurse is demonstrating the assessment phase of the nursing process? A.The nurse who observes that the client's pain was relieved with pain medication B. The nurse who turns the client to a more comfortable position C. The nurse who ask the client how much lunch he or she ate D. The nurse who works with the client to set desired outcome goals

C. The nurse who ask the client how much lunch he or she ate Rationale: Assessment involves collecting, organizing, validating, and documenting data about a client. Option 1 represents the evaluation phase. Option 2 represents the implemention phase. Option 4 represents the planning phase.

Which of the following items of subjective client data would be documented in the medical record by the nurse? A. Client's face is pale B. Cervical lymph nodes are palpable C. Nursing assistant reports client refused lunch D. Client feel nauseated

D. Client feel nauseated Rationale: Subjective data includes the client's sensations, feelings, and perception of health status. Subjective data can only be verified by the affected person. Options 1, 2, and 3 represent objective data that can be detected by the nurse or measured against an accepted norm.

A client on the nursing unit is terminally ill but remains alert and oriented. Three days after admission, the nurse observes signs of depression. The client states, "I'm tired of being sick. I wish I could end it all." What is the most accurate and informative way to record this data in a nursing progress note? A. Client appears to be depressed, possibly suicidal B. Client reports being tired of being ill and wants to die C. Client does not want to live any longer and is tired of being ill D. Client states, "I'm tired of being sick. I wish I could end it all."

D. Client states, "I'm tired of being sick. I wish I could end it all." Rationale: Subjective data includes thoughts, beliefs, feelings, perceptions, and sensations that are apparent only to the person affected and cannot be measured, seen, or felt by the nurse. This information should be documented using the client's exact words in quotes. The other options indicate that the nurse has drawn the conclusion that the client no longer wishes to live. From the data provided, the cues do not support this assumption. A more complete assessment should be conducted to determine if the client is suicidal.

In giving a change-of-shift report, which type of client information communicated by the nurse is most appropriate? A. Vital signs are stable B. Client is pleasant, alert, and oriented to time, place, and person C. The chest x-ray results were negative D. Client voided 250 mL of urine 2 hours after the urinary catheter removal

D. Client voided 250 mL of urine 2 hours after the urinary catheter removal Rationale: A change-of-shift report should include significant changes (good or bad) in a client's condition. The information should be accurate, concise, clear, and complete. Options 1 is vague and options 2 and 3 are normal data and are therefore of lesser importance to convey in the change-of-shift report.

The nurse would write which of the following outcome statements for a client starting an exercise program? A. Client will walk quickly three times a day B. Client will be able to walk a mile C. Client will have no alteration in breathing during the walk D. Client will progress to walking a 20-minute mile in one month

D. Client will progress to walking a 20-minute mile in one month Rationale: Outcome statements must be written in behavioral terms and identify specific, measurable client behaviors. They are stated in terms of the client with an action verb that, under identified conditions, will achieve the desired behavior. They should also be realistic and achievable.

While assisting a client from bed to chair, the nurse observes that the client looks pale and is beginning to perspire heavily. The nurse would then do which of the following activities as a reassessment? A. Help client into the chair but more quickly B. Document client's vital signs taken just prior to moving the client C. Help client back to bed immediately D. Observe client's skin color and take another set of vital signs

D. Observe client's skin color and take another set of vital signs Rationale: Assessment is ongoing throughout the nurse-client relationship. During re-assessment, the nurse collects additional data to help evaluate the status of problems or identify new problems. Options 1, 2, and 3 are interventions.

The nurse evaluates the client's progress and determines that one of the nursing diagnoses on the client's care plan has been resolved. How should the nurse document this so that it is best communicated to the healthcare team? A. Use Liquid PaperTM to "white out" the resolve diagnosis on the care plan B. Recopy the care plan without the resolve diagnosis C. Write a nursing process not indicating that the outcome goals have been achieved D. Draw a single line through the diagnosis on the care plan and write the nurse's initials and date

D. Draw a single line through the diagnosis on the care plan and write the nurse's initials and date Rationale: To discontinue a diagnosis once it has been resolved, cross it off with a single line or highlight it, then write initials and date. Some agency forms may require the nurse to put date and initials in a "Date Resolved" column. Using Liquid PaperTM is not a legal way to amend client records. Outcome goals that have been met and nursing diagnoses that have been resolved should be documented on the care plan. A progress note should also be written, but a single note may not be read by all health team members.

Twenty minutes after administering pain medication to the client, the nurse returns to ask if the client's level of pain has decreased. The nurse documents the client's response as part of which phase of the nursing process? A. Diagnosis B. Planning C. Implementation D. Evaluation

D. Evaluation Rationale: Evaluating is the process of comparing client responses to the outcome goals to determine whether, or to what degree, goals have been met. Diagnosing identifies health problems, risks, and strengths. Planning is the formulation of client goals and nursing strategies (interventions) required to prevent, reduce, or eliminate the client's health problems. Implementing is carrying out or delegating the nursing interventions.

For the nursing diagnostic statement, Self-care deficit: feeding related to bilateral fractured wrists in casts, what is the major related factor or risk factor identified by the nurse? A. Discomfort B. Deficit C. Feeding D. Fractured wrists

D. Fractured Wrists Rationale: The etiology or related factors of a nursing diagnostic statement define one or more probable causes of the problem and allow the nurse to individualize the client's care. In this case, the fracture is the cause of the client's feeding problem.

The rehabilitation nurse wishes to make the following entry into a client's plan of care: "Client will reestablish a pattern of daily bowel movements without straining within two months." The nurse would write this statement under which section of the plan of care? A. Nursing diagnosis/problem list B. Nursing orders C. Short-term goals D. Long-term goals

D. Long-term goals Rationale: Long-term goals describe changes in client behavior expected over a time frame greater than one week. They are usually designed to restore normal functioning in a problem area and are helpful to other healthcare workers who care for the client, often in a variety of settings.

A nurse explains to a student that the nursing process is a dynamic process. Which of the following actions by the nurse best demonstrates this concept during the work shift? A. Nurse and client agree upon health care goals for the client B. Nurse reviews the client's history on the medical record C. Nurse explains to the client the purpose of each administered medication D. Nurse rapidly reset priorities for client care based on a change in the client's condition

D. Nurse rapidly reset priorities for client care based on a change in the client's condition Rationale: The nursing process is characterized by unique properties that enable it to respond to the changing health status of the client. Options 1, 2, and 3 are appropriate nursing care measures, but do not demonstrate the dynamic nature of the nursing process.

The nurse has documented the following outcome goal in the care plan: "The client will transfer from bed to chair with two-person assist." The charge nurse tells the nurse to add which of the following to complete the goal? A. Client behavior B. Conditions or modifiers C. Performance criteria D. Target time

D. Target time Rationale: The outcome goal does not state the target timeframe for when the nurse should expect to see the client behavior ("transfer"). The condition or modifier is present ("with two assists"). The performance criterion is "from bed to chair."


Conjuntos de estudio relacionados

Ivy Tech Online Biology: Session 7

View Set

Doing social psychology research 2

View Set